Форум » Теория » Критерий выбора рабочей точки » Ответить

Критерий выбора рабочей точки

B_Aleko: Приветствую всех. Решил заняться сборкой ламповых усилителей вплотную. Ранее, лет 10-12 назад, был небольшой опыт сборки 5 усилков..., в подробности их работы, особо не вникал, но солидные люди интересовались, слушали... и продал. Но несколько месяцев всё прочитываю литературу, и никак не могу выяснить способ выбора рабочей точки "с нуля", то есть: взять лампу, её ВАХ и... поставить точку РТ. И меня смущает тот факт, что во многих пересмотренных схемах, РТ всегда располагается ниже тока, который заявлен в параметрах ламп (подробнее объясню в обсуждении). Это еще не все. Будет масса интересных "моих" вычислений по этой теме и другим. Прошу подробно объяснить данное вычисление или принцип, отнестись без скептицизма и как к новичку. Спасибо.

Ответов - 246, стр: 1 2 3 4 5 6 7 8 9 10 11 12 13 All

ALSS: Один из критериев - непревышение мощности рассеяния на аноде (и сетках) лампы - это к "ниже тока, который заявлен в параметрах ламп". Главные критерии - требуемый уровень выходного сигнала (и входного тоже) и минимум искажений. Искажения считаются или в симуляторах или методами трех/пяти ординат. Поэтому ищется точка, влево-вправо от которой (по прямой нагрузки, которая поворачивается для удовлетворения этим требованиям) выходные напряжения одинаковы при одинаковых изменениях входного напряжения (справедливо для триодов). Или Вы хотите поделиться своим методом нахождения РТ, судя по "Будет масса интересных "моих" вычислений"? Не хочу показаться грубым, но почему-то сразу вспомнилось выражение Алекса Торреса "Прежде чем сделать открытие загляни в учебник"...

Stan Marsh: О каких каскадах речь? Предварительных, оконечных? Трансформаторных, резистивных?

B_Aleko: Речь пока о триодах в преде. Но начну с того, что анодное сопротивление - это прямая, соединяющая две точки: питания и тока. Но возможно ли его вычислить из РТ? И можно ли подробнее про выбор.


Stan Marsh: B_Aleko пишет: анодное сопротивление... возможно ли его вычислить из РТ? Что? Как это?

B_Aleko: Позже нарисую и покажу на ВАХ Вообще-то, я хотел раскрытого объяснения выбора Рабочей Точки. Не по напряжению, не по току, не по смещению, а, сперва по визуальному взгляду на ВАХ. и при доступности относительно высокого анодного напряжения. Зная из характеристик средние значения, можно варьировать (в разумных пределах) токами, напряжениями, смещением. но при этом соблюсти баланс разностей всего перечисленного. Как-то так. При расчете из РТ, а не из крайних точек, (по стандартным формулам Ома), есть много интересных выводов... Прошу обойтись без цитирования кого-либо и адекватно общаться. Ведь способы вычисления одни.

Пермяк: B_Aleko пишет: И меня смущает тот факт, что во многих пересмотренных схемах, РТ всегда располагается ниже тока, который заявлен в параметрах лампПохоже, Вы сравнивали режимы с данными из даташитов на лампы, под заголовком Номинальные электрические данные. Следует знать, что это не данные для РТ какого-либо каскада, а данные КОНТРОЛЬНОЙ точки. Именно в этой точке проверяют параметры ламп (Ri, S, µ) на заводе-изготовителе на соответствие указанным допускам. Не соответствует - брак!, поставке не подлежит. И потребитель, получающий лампы на своём предприятии, проверяет их параметры строго в этой же точке. Не укладываются в допуски - брак, вернуть изготовителю для замены, а не пожелает - предъявить ему штрафные санкции!подробнее объясню в обсужденииИзлагайте, попытаемся Вас понять ЗЫ. Такое пожелание. Если будете выкладывать свои мысли просто текстом, понять Вашу концепцию нам будет трудновато. Сопровождайте Ваши мысли рисунками, вот Вам подсказка: http://hiend.borda.ru/?1-16-0-00000106-000-0-0-1460610681

B_Aleko: Хорошо. Чуть позже нарисую. НО, всё-таки, хотелось бы знать о способах выбора РТ!?...

Stan Marsh: B_Aleko пишет: НО, все таки хотелось бы знать о способах выбора РТ!?... Открываем учебник, там всё описано. Если сильно лениво, то http://www.radiolamp.ru/shem/unch/112/

B_Aleko: Вот в том то и дело: "Отложив на оси напряжений величину напряжения источника питания (А), на оси токов величину (В):" А как этот источник напряжения-то выбрать? может кто захочет и 100 вольт. или величину тока загнать вверх? Тогда как? Я имею в виду критерий ВЫБОРА не относительно этих величин! Может надо как то по другому объяснить? как?

oleg_s: B_Aleko Берите схемы удачных усей. И изучайте на ВАХ режимы, что люди выбрали. Это и есть ответ на ваши вопросы. И никого не надо будет упрашивать. Зато вопрос можно задать реальный: " Ты почему такую точку выбрал в этом месте?"

B_Aleko: И я про то же! Только никак в толк взять не могу, почему РТ внизу ВАХ?

oleg_s: B_Aleko, В 90% случаев - это компромисс с питанием. Нормальные герои лампу жарят и 500 вольт дают. Другие - используют трансформаторную нагрузку, и т.д. В каждом техническом решении свои недостатки и преимущества.

B_Aleko: Однако. Вот график. что можно по нему сказать с ваших точек зрения? http://shot.qip.ru/00Qiah-1YLAzZMmv/

oleg_s: B_Aleko, Ну... На приложенном примере видно, что разработчик задался напряжением питания анодной цепи 200 Вольт. Судя по наклону нагрузочной прямой, анодная нагрузка 15 кОм. Смещение сетки лампы -1,5В. Разработчика устроил коэффициент усиления по напряжению Ку=24. Исходя из Ri=5.8кОм, его устроило выходное сопротивление 15*5.8/(15+5.8) = 4.2 кОм, что достаточно при средних значениях входной ёмкости выходного каскада. ЗЫ Мог ошибиться в расчетах.

ALSS: Сопротивление анодной нагрузки 15 кОм (200В/13 мА). Напряжение источника питания выбрано 200В. При автоматическом смещении 198,5 В. Но никто эти десятые доли и даже 2-3 вольта анодного питания не учитывает: лампы имеют больший разброс. Поэтому считаем, что напряжение источника питания 200В. Сопротивление анодной нагрузки 15 кОм (200В/13 мА). Напряжение смещения около 1,5В.. Каскад линеен до выходного напряжения ±35 В, выходное сопротивление 4,8 кОм (параллельное соединение внутреннего сопротивления лампы в рабочей точке 7,1 кОм и нагрузки 15 кОм, без учета шунтирования входным сопротивлением следующего каскада. Но возникают вопросы: а сколько надо на выходе? а что подается на вход? Я, например, применяю лампы со смещением 6-10 вольт на входе и вольт 20-30 в драйвере - голова не болит по поводу залезания сигналом на границы линейного режима. PS. oleg_s чуток опередил, но - два подсчета дали совпадающие результаты.

Сергеев Сергей: Немаловажное значение имеет выбор напряжения смещения, чтобы от уровня сигнала мгновенное значение напряжения на сетке не было ниже 0. Иначе сеточные токи и соответственно искажения. Если пик сигнала 1 вольт, то смещение выбирают не менее 1,5-2 вольта. Я по крайней мере начинаю с этого.

B_Aleko: Сергеев Сергей пишет: Если пик сигнала 1 вольт, то смещение выбирают не менее 1,5-2 вольтаВот это уже близко. Приму к сведению. oleg_s пишет: Исходя из Ri 5.8К...Не совсем понял, как Ri получилось 5,8 кОм? Вроде там мю=24 и S=1,6 У меня так: мю=24,7 S=1,6 Ri=15,4 Ra=15,4 Rk=260-280 Ом Uс (РТ)=-1,4 Положительная волна по напряжению немного меньше отрицательной. Или не так считаю, формулы-то одни и те же. Вопрос: возможно ли так, что внутреннее сопротивление лампы равно анодному?

oleg_s: B_Aleko пишет: Не совсем понял, как Ri получилось 5,8? Вроде там м=24 и S=1,6http://istok2.com/data/575/ Мю = 38±6, S = 5.9±1.8

B_Aleko: oleg_s, так это - паспортные данные, а я - по расчету в данном рисунке. Еще вопрос. При входном сигнале с амплитудой в 1В, по данному рисунку, сколько выходное? что-то я никак. ---------------------- В выходном каскаде хочу поставить 6П13С (их у меня аж 10 шт. все новые) или 6П31С (4 шт.). Как можно согласовать с 6Н3П, с учетом, что положительная волна амплитуды немного меньше отрицательной? И хотелось бы в 6П13С смещение от -4 до -8V P.S. Есть еще вариант. имеются 6Ж32П - 12 шт. тож новые. но в пентодах пока не силен. вместо 6Н3П?

ALSS: B_Aleko пишет: я - по расчету в данном рисунке. 1. Параметры лампы вычисляют по её ВАХ в окрестностях выбранной РТ, и желательно (т. к. параметры - величины дифференциальные) - при минимальных от нее отклонениях. По приведенному рисунку: Ri=∆Up/∆Ip=(136-102)/(7,8-3,0)=34/4.8=7,1 кОм S=∆Ip/∆Ug=(7,8-3,0)/(2-1)=4,8/1=4,8 мА/В mu=∆Up/∆Ug=(136-102)/(2-1)=34/1=34 (все численные значения округлены ввиду графического исходного материала). А коэффициент передачи (коэффициент усиления) при нагрузке 15 кОм равен: Ку= (130-106)/(2-1)=24 (без учета шунтирования входным сопротивлением последующего каскада) 2. Для 6П13С в триодном включении, в качестве драйвера гораздо лучше применить 6Ж4, 6Ж8 (обе в триоде) или 6Г2 - вообще хорошо. B_Aleko пишет: Есть еще вариант. имеются 6Ж32П - 12 шт. тож новые. но в пентодах пока не силен. вместо 6Н3П? Не надо 6Ж32П с током анода всего 1-2 мА нагружать на 6П13С, ранний завал ВЧ обеспечен.

B_Aleko: Ri=∆Up/∆Ip=(136-102)/(7,8-3,0)=34/4.8=7,1 кОмЭто взято при дельтаUc с разницей в 1В? но по формулам и из рисунка при дельтаUc 3В: (152-78)/(7,8-3,0)=74/4,8=15,4 ! в чем разница таких расчетов? 6Н3П выбрал по причине большого количества 46шт. Есть 6н1, 6н2, 6н23, 6ф1, 6ф12, 6ж1,2,3,4,5,9(63шт),10,11,32,38,44,49,51,52; 6к4, 6к13, 6с4п(2шт), 6в2п других нет.

Пермяк: B_Aleko пишет: в чем разница таких расчетов?Дело в следующем. ∆U - некоторое изменение напряжения. Можно взять и 1 В, и 3 В. Но по теории, для определения дифференциальных величин S, Ri, Мю мы должны брать в РТ бесконечно малые приращения напряжений и токов: dU и dI, тогда будет максимальная точность результатов вычислений. Но бесконечно малые невозможно отложить на графике, и приходится использовать те кривые ВАХ, что есть на графике. Чем меньше выбранные "дельты", тем лучше, а значит, лучше выбирать соседние линии из семейства ВАХ. Опять же, если имеющийся график семейства ВАХ имеет удобный масштаб, чтобы легче было определять отрезки, спроецированные на его оси. -------------------- to All Для тех, кто пишет, что 6н3п - кривовата, предлагаю посмотреть ВАХи, практически снятые Клаус'ом на характериографе: Позже Алекс Рощупкин снял ВАХи 6н3п с реальной лампы "по точкам", вот что получил: Видим , что 6н3п - самый обычный, достаточно линейный триод.

B_Aleko: Вот, вроде запутался. Значит, при полном размахе величин "дельта" имеем максимальные выходные параметры? и для каждого режима усиления все справедливо?

Пермяк: "Дельтами" - называют маленькие кусочки линии или координат их крайних точек. При полном размахе имеем уже не "дельты", а АМПЛИТУДЫ сигнала.

B_Aleko: Пермяк пишет:"Дельтами" - называют маленькие кусочки линии или координат их крайних точек. При полном размахе имеем уже не "дельты", а АМПЛИТУДЫ сигнала.тогда где эти дельты на графике?

ALSS: B_Aleko пишет:тогда где эти дельты на графике?Минимально возможные видимые на графике дельты возьмите из моего сообщения №1092 и найдите их на приведенном Вами графике (как, Вы этого еще не сделали?).но по формулам и из рисунка при дельтаUc 3В: (152-78)/(7,8-3,0)=74/4,8=15,4! Если брать смещение 0 и минус 3 В, то (174-62)/(15-1)=108/14= 7,7 кОм! Т. е. при расчете по амплитудам вместо дельт, как справедливо указал Пермяк, значение изменяется, но немного (зависит от лампы, конечно - у линейной лампы это изменение меньше) и не в разы... PS. При использовании формул и рисунков не забывайте о физическом смысле рассчитываемого параметра. Вдогонку - 6Ж51П, 6Ж52П в триоде - прекрасные лампы! 51-ая к тому же не микрофонит.

B_Aleko: ALSS пишет: Если брать смещение 0 и минус 3 В, то (174-62)/(15-1)=108/14= 7,7 кОм! вот окончательно запутался. напишите формулу, а с цифрами разберусь

oleg_s: B_Aleko , да просто все S = LIM ( dI(Uo,Io)/dUg(Uo,Io) ) при dUg -> 0, Ua= const

B_Aleko: oleg_s пишет:Да просто все спасибо. теперь все стало на места. То, что я считал, что Rа=200в/13мА=15,4 кОм, можно делить на 2, ввиду разницы напряжения смещения в 1В. а значит 15,4/2=7,7. но здесь мне кажется более точнее. Может, использовать координаты самой точки? ведь полученные данные верны для полупериода. Тогда при положении в РТ получим еще более точное значение: берем Uo=116, Io=5,4 получаем: Ri=(Uo/Io)/2=(116/5.4)/2=10.75 Это вернее для случая при отсутствии сигнала на входе и относительно небольшого сопротивления цепи питания. По расчетам выходит, что чем больше входной сигнал - амплитуда, тем меньше внутренне сопротивление лампы. вывод - считать при 0 амплитуды, в РТ, и при макс. значении.

oleg_s: B_Aleko, Я дал формулу крутизны S. На графике для определения S дельта тока берется для двух соседних ветвей ВАХ по вертикали, т.е. при неизменном Ua. Для определения Ri берется касательная к одной ветке ВАХ в выбранной точке (РТ), там дельты анодных характеристик, в пределе получается производная, графическое отображение этой производной - это котангенс угла наклона касательной к кривой в данной точке.

B_Aleko: oleg_s, хорошо когда знаешь расчет и оперируешь научными терминами. а как быть, если только начинаешь? а попроще...........

ALSS: B_Aleko, да не запутывайте Вы себя формулами, в особенности если физический смысл их ускользает. Потрудитесь построить указанные "дельты" и "амплитуды" и вникнуть в соотношения величин. B_Aleko пишет: Uo=116, Io=5,4 получаем: Ra=(Uo/Io)/2=(116/5.4)/2=10.75 Это статическое сопротивление лампы в данной рабочей точке, да и то неправильно рассчитанное, а искомое Ri - динамическое сопротивление.по расчетам выходит, что чем больше входной сигнал - амплитуда, тем меньше внутренне сопротивление лампы. вывод - считать при 0 амплитуды, в РТ, и при макс. значении. Внутреннее сопротивление лампы от входного (и выходного тоже) сигнала не зависит - это-то Вы понимаете? Зачем считать при нуле, если надо усиливать сигналы при отрицательном напряжении на управляющей сетке (а, забыл, под фонарем светлее...)?! PS. А если еще и подбросить классическое "Жарьте Ваши триоды!", так вообще крышу снесёт, наверное...

B_Aleko: Спасибо всем, думаю что с теорией вкусную кашу не сваришь, хрустеть стеклами будет. надо на практике либо не до жарить, либо забыть до черного дыма. на ветке останусь. может по формулам всем определюсь. в книгах одно, а на деле иначе.

Пермяк: B_Aleko пишет: с теорией вкусную кашу не сваришь,Не теория виновата, а те, кто косноязычно пытается её излагать! Всегда поражался, насколько плохо "технари" излагают свои мысли на форумах... Должен сказать, что график ВАХ, на котором Вы делаете расчёт - неудачный. Эти ВАХ, когда-то очень коряво снятые, перекочевали практически во все справочники, и при первом взгляде на них просто отталкивают от применения 6н3п в звуковой аппаратуре. Между тем, ещё Ю.Макаров, применявший эту лампу, писал на форуме Аудиопортал, что ВАХи эти - неправильные. Я выше (пост 3972) привёл ВАХ, снятые Клаус'ом цифровым характериографом,, а также ВАХ от Алекса Рощупкина.

B_Aleko: Пермяк пишет: Не теория виновата, а те, кто косноязычно пытается её излагать! Всегда поражался, насколько плохо "технари" излагают свои мысли на форумах... вообще-то, я имел в виду, что на одной теории сыт не будешь, без масла с сахаром в виде практики, не вкусно будет. чтобы быть более обстоятельным, на рисунке добавил формулы которые вычитал в книгах. и график чертил по описаниям многих книг. от них и отталкиваюсь. http://shot.qip.ru/00Qiah-2YLAzZMmP/

Пермяк: B_Aleko пишет: на рисунке добавил формулы Каждая из этих формул имеет смысл только тогда, когда рядом с ней написано условие: при неизменной такой-то величине. ЗЫ. Я уже начал готовить картинки, чтобы выложить здесь в теме. Ваш рисунок слишком труден.

B_Aleko: может это поможет: http://shot.qip.ru/00Qiah-3YLAzZMmQ/ картинка прозрачная, поверх первой положить и видно будет. этот треугольник и есть алгоритм выбора РТ. главное, что бы вершина и половина гипотенузы совпадали с линией смещения сетки. а равносторонний треугольник можно вращать до выбора оптимальной точки. ведь как его не крути, то дельты U и I всегда будут одинаковы. прошу не осуждать, если неправильно что то делаю, книги хорошо но люди - лучше. вот решил на графике "Клаусмобиль" применить треугольник. особо не старался. (не осуждайте) http://shot.qip.ru/00Qiah-2YLAzZMmR/ На графике по расчетам мю 24-25, крутизна почти 1. сопротивление в аноде 27 кОм, катодный 290 Ом, макс. мощность анодного резистора 1,4 Вт.

ALSS: Не всегда получается выбирать нагрузку по перпендикуляру асимптоте зависимости анодного тока от анодного напряжения при заданном напряжении смещения, да и не всегда это оптимальное решение. В линейных лампах хоть как клади нагрузку - будет все симметрично, в нелинейных - хоть тресни и ищи стабильные точки по Карпову, толку не будет (речь идет о больших сигналах).

Пермяк: to B_Aleko Предлагаю Вам более удобный (как мне кажется) пример для рассмотрения метода определения параметров триода. Пока ознакомьтесь, о параметрах - чуть позже. Можно определить коэффициент усиления Ку такого каскада. Мы видим, что если амплитуда сеточного напряжения равна 1 В, амплитуда анодного напряжения составляет (153-95)/2=29 Тогда Ку= Ua/Uc=29/1= 29. (где Ua и Uc - амплитуды анодного и сеточного напряжений).

B_Aleko: Пермяк, а думаете есть смысл в уменьшении размаха сеточного напряжения? а можно ли тогда ваш принцип расчета? пока подумаю над рисунком.

Пермяк: B_Aleko, я не уменьшил размах сеточного напряжения. Для каждой полуволны - по 1 вольту запас. Вам ранее предлагали смещение -2 В, Вы согласились, я это и отразил. А коэффициент усиления практически не зависит от того, какой величины сигнал подаётся на сетку.

B_Aleko: Пермяк пишет: коэффициент усиления практически не зависит от того, какой величины сигнал подаётся на сетку. тогда как это ВСЁ рассчитывать? столько инфы в нете, много читаю, но в толк не возьму. и как выбрать правильные расчеты? ЗЫ. Забыл сказать. ПРАКТИЧЕСКИЕ работы не скоро смогу воплотить. так как многое сгорело в доме (оч. давно), и приборы. а вот сейчас решил заняться из того что есть. потому и начал с теории. в которой надо разобраться, палить и фейерверки утраивать на съемной кв. не хочется.

oleg_s: B_Aleko пишет:тогда как это ВСЁ рассчитывать? забыл сказать. ПРАКТИЧЕСКИЕ работы не скоро смогу воплотить. Есть подозрение: вам надо микрокап освоить - будете рассчитывать в уме и на машине и сравнивать.

B_Aleko: Пермяк пишет: Ку - частное от деления двух величин. Если сеточная амплитуда возрастёт в два раза, то и анодное - почти во столько же раз, и частное от деления останется таким же. значит, в выше приведенном графике, при увеличении сеточного на 1 В, то вместо 95В будет 80, а вместо 153 - 165? oleg_s пишет: микрокап освоить ага. знаю такую. если бы трое детей дали возможность! а так - на ходу все считаю.

oleg_s: B_Aleko пишет: если бы трое детей дали возможность! Учите с ними! Они быстрее освоят. Зуб даю. Моя 8ми-летка за два тыка научилась у папы в смартфоне фотки смотреть.

Пермяк: B_Aleko пишет: значит, в выше приведенном графике, при увеличении сеточного на 1 В, то вместо 95В будет 80, а вместо 153 - 165? Да. Примерно так.

омельян: B_Aleko пишет:тогда как это ВСЁ рассчитывать? http://zzxm.narod.ru/ http://www.oldradioclub.ru/catalog/book_links_k.html С.Н.Кризе "Усилительные устройства", 1968 год А так же: http://radiolamp.ucoz.ru/load/literatura/lampovye_usiliteli_moshhnosti_v_rezhime_klassa_a/5-1-0-80 - скачать. Последняя ссылка - там всё просто и понятно. Внимательно прочтите. Проще не бывает.

B_Aleko: спасибо. Кризе - обстоятельно, но сразу с формул. нашел в нете книгу: Гершунский Б. С. Основы электроники, 1977 г., и немного сканов из его книги 1966г. глава про лампы, там вообще по полочкам каждый параметр.

Пермяк: B_Aleko Мой метод определения параметров лампы в данной точке отличается от некоторых распространённых, с построениями треугольников. Считаю, что математически он более правильный и простой. Внутреннее сопротивление (Ri) лампы вычисляем по формуле: Ri = µ/S = 38/5,5=6,9 кОм

B_Aleko: Пермяк пишет: Мой метод определения параметров лампы вот спасибо. Не мог сообразить вычисление дельты тока. брал с наклонной нагрузки анодного. oleg_s пишет: будет делать с крутизной лампы? еще думаю. но смутные догадки, что величина влияет на полосу пропускания

B_Aleko: Вопрос: если, например, Ri=7к, Ra=27к, то тогда для чего эта разница в сопротивлениях?

B_Aleko: форум поможет в окончательном вычислении результатов? на графике есть точки. как все таки правильно высчитываются "дельты"? (график с хорошим разрешением) http://shot.qip.ru/00Qiah-1YLAzZMn4/ можно ли для сверки результатов, рассчитать параметры по графику и приписать их к графику? и + входное и выходное напряжение. только с питанием пока засада. в наличии пока 300В основное и, что могу для преда - это 270В. благодарен, кто отзовется в мои трудные минуты...

ALSS: Пермяк пишет: Мой метод определения параметров лампы в данной точкеЭто и есть классический способ определения параметров! И без всяких к делу не относящихся треугольников и параллелепипедов.

Пермяк: B_Aleko пишет: вычисление дельты тока. брал с наклонной нагрузки анодного. Эти параметры никак не связаны с нагрузкой. Кроме того, в теории сказано: мю=dUa/dUc при неизменном токе анода Ia, то есть, вспомогательный отрезок АВ должен быть горизонтальным. S = dIa/Uc при неизменном напряжении Ua, то есть, вспомогательный отрезок СD должен быть вертикальным. мю=dUa/dUc при неизменном токе анода Ia, то есть, вспомогательный отрезок должен быть горизонтальным. Наклонные линии не нужны.что величина крутизны влияет на полосу пропускания Значение S может пригодиться для выбора шунтирующего катодный резистор конденсатора (при автосмещении). То есть, действительно, S косвенно влияет и на полосу частот. Собственно, как и другие параметры - мю и Ri. если Ri=7к, Ra=27к, то тогда для чего эта разница в сопротивлениях? Для триода - чем больше Ra по отношению к Ri, тем меньше искажений и выше Ку. В идеале хорошо бы, если бы Ra было равно бесконечности

B_Aleko: Вот, вроде так: а усиление каскада?

Пермяк: B_Aleko пишет: вопрос: для чего надо увеличивать анодное сопротивление для триода (2-10Ra) и уменьшать для пентода (0.1-.02Ra)?Про триод я Вам писАл чуть выше (мой пост 3993): "Для триода - чем больше Ra по отношению к Ri, тем меньше искажений и выше Ку. В идеале хорошо бы, если бы Ra было равно бесконечности" Для пентода специально уменьшать Ra не надо, его ВАХи таковы, что Rа равное (0,1-0,2) от Ri получается среднестатистически где-то в этих пределах.

Пермяк: B_Aleko пишет: а усиление каскада?Ку= Ua/Ug где: Ua - амплитуда анодного напряжения, Ug - амплитуда сеточного напряжения. Попробуйте сосчитать сами, в качестве подспорья выкладываю график. Кривые (синусоиды) напряжений при расчёте, разумеется, не рисуют, это я подрисовал Вам для лучшего понимания.

B_Aleko: Пермяк Спасибо огромное за помощь и терпение. я бы еще долго разбирался. По этому графику при входном сигнале амплитудой 1В: Ku=(165-90)/2=37,5.. С праздниками всех!

Пермяк: С праздником, коллега! Снимать проекцию на ось напряжений надо с точек на нагрузочной прямой. Тогда, при амплитуде на сетке Ug.m=1В, амплитуда анодного напряжения будет равна: Ua.m=(155-100)/2=27,5 В и усиление будет равно Ку=27,5/1=27,5. А при амплитуде на сетке Ug.m=2В (как на рисунке): Ua.m=(180-70)/2=55 В Ку=55/2=27,5 Ку с ростом входного сигнала не изменяется, о чём мы писали ранее.

B_Aleko: Пермяк пишет: Снимать проекцию на ось напряжений надо с точек на нагрузочной прямой.! только сейчас увидел. Извиняюсь очень, не те цифры вставил, это мю. эх, как эт я не аккуратно...

B_Aleko: приветствую всех. у меня засада с пентодами и луч. тетродами. пентодов оч. много. на выход хочу поставить 6п13с, пока однотакт, чтобы разобраться в расчетах. Только почему-то встречаю его смещение в -45В. а на ВАХ только до -16В. скачал книгу "Справочник электровакуумным и полупроводниковым приборам А.М.Бройде 1962", там есть ВАХ многих ламп. но таких параметров не нашел. подскажете по ВАХ-ам?

омельян: B_Aleko пишет: пока однотакт Будете делать в триоде? Или пока - разобраться? Ia=50...60 mA; Ua=325...330 V. Так же необходимо учесть падение напряжения питания на первичной обмотке вых. трансформатора. B_Aleko пишет: в триодном не могу найти эти ВАХ http://radikal.ru/big/3f5650b6fc3248609e46f6d39d5f27d8] Я пользовался этой ВАХ.

B_Aleko: омельян пишет: Будете делать в триоде? Или пока - разобраться? пока хочу разобраться в расчете по ВАХ в триодном и тетродном включении. Но в триодном не могу найти эти ВАХ. Расчеты есть, но без наглядного графика - никак!

Пермяк: ВАХи Александра Рощупкина (слегка доработал, думаю, А.Р. не рассердится.) Rэа - резистор, соединяющий экранную сетку с анодом.

B_Aleko: Пермяк пишет: ВАХи Александра Рощупкина Спасибо. А где взять эти ВАХи Александра Рощупкина? нашел Бройде 1962 года.

Stan Marsh:

B_Aleko: Stan Marsh, огромнейше благодарен! А какой ток анода лампы? в параметрах написано только: в импульсе 200мА?

Пермяк: B_Aleko пишет: А где взять эти ВАХи Александра Рощупкина?У Александра был сайт на pochta.ru, где он и выкладывал снятые им характеристики. Примерно год назад почта.ру приказала долго жить, и новый владелец не торопится восстановить все сайты. Ток, который длительно выдерживает лампа, легко определить по кривой (гиперболе) Pa+Рэ на графике для выбранного Ua.

B_Aleko: спасибо, кто откликнулся. хотел бы изучить расчет тетродов и пентодов по графику, для драйверов и выходного. а в однотакте проверить и триодное и пентодное включение 6П13С.

aur_100: 6п13с в триоде - моя любимая (из дешевых) лампа. 240в, 60мА, смещение ~-25в, естевственно в триоде.. В тетроде НЕ ставлю... Я брал Ra=5кОм, мощность меньше, и гармоники тоже. Кому нужней мощность, можно Ra=3кОм. Наши лампы спокойно рассеивают до 18 Вт...

B_Aleko: aur_100, мне кажется, что -25 - многовато. А с учетом того, что пред рассчитываю на Ку=25-28, то по графику должно совпадать вход=26В. http://shot.qip.ru/00QjE0-4131FKD8B2/

aur_100: 6н3п не даст нужного Ку... Я ставил 6ж4 в триоде, вых. 6п13с.

Пермяк: B_Aleko, Вы неправильно ставите РТ. Для каскада усиления мощности с выходным трансформатором построение ведётся не так, как для каскада с резистором в аноде. Вот пример:

B_Aleko: Пермяк и по этому графику выходит, что питание=512В???

Stan Marsh: B_Aleko пишет: по этому графику выходит, что питание=512В??? Нет, по этому графику оно равно +300В.

B_Aleko: Stan Marsh пишет: Нет, по этому графику оно равно +300В. так это справедливо для двухтакта? иначе как быть с правой полуволной сигнала?

Stan Marsh: B_Aleko пишет: это справедливо для двухтакта? Это справедливо для трансформаторного однотактного каскада. как быть с правой полуволной сигнала? Пользоваться. ...

Пермяк: B_Aleko, для пояснения, как образуется положительная полуволна в трансформаторном каскаде, привожу выдержку из статьи: Михаил Смелянский. К вопросу о выборе минимально допустимой индуктивности..." Как известно, положительная полуволна выходного сигнала в однотактном каскаде формируется засчёт энергии, накопленной в индуктивности засчёт протекающего через нее тока покоя. Постулат: условием неискаженной передачи положительной полуволны выходного сигнала является выполнение баланса : Qn ≥ Qi где: Qn - энергия, накопленная в катушке (Дж). Qi - энергия положительной полуволны (Дж); Таким образом, для неискаженной передачи положительной полуволны сигнала, индуктивность должна содержать достаточное количество энергии, определяемое формулой: Qn =(Iр2 * L) / 2 где: Ip - анодный ток покоя L - индуктивность первичной обмотки трансформатора.

B_Aleko: Пермяк пишет: Для каскада усиления мощности с выходным трансформатором построение ведётся не так, как для каскада с резистором в аноде. Вот пример: С праздником!!! По этому примеру почти понятно. Получается, что на первичке трансформатора почти нет падения питающего напряжения до значения в РТ. Поэтому РТ выбирается исходя от анодного. Тогда положение РТ берется по току покоя и напряжению смещения сетки. Вопрос: зачем брать смещение, заведомо больше? такого расчета нигде не нашел. Но это в триоде. А как на пентодах (распространенных 6п14п, 6п15п, 6п18п, 6п43п (достались из бывшей мастерской)? Точно так же?

Stan Marsh: Смещение выбирается так, чтобы не была превышена максимально допустимая мощность анода. Т.е., РТ должна лежать чуть ниже кривой максимально допустимой мощности, рассеиваемой на аноде. Или, при отсутствии такой кривой на графике, должно выполняться условие: Ua0*Ia0 ≤ Pa.max.доп.

B_Aleko: Stan Marsh пишет: Смещение выбирается так, чтобы не превысить максимально допустимую мощность анода. и как это по графику выбирается? там ведь пойдут области искажений!?

Stan Marsh: B_Aleko пишет: как это по графику выбирается? Посмотрел в паспорт, увидел 20Вт, При выбранном анодном 300В, ток покоя выбираем 0,9*20/300=60мА смотрим, что этому соответствует смещение -20В такое и обеспечиваем. (примерно так)

B_Aleko: Stan Marsh пишет: смотрим, что этому соответствует смещение -20В теперь понятно. а дальше как? по графику: http://shot.qip.ru/00Qk6h-6d3R3GHLx/

Stan Marsh: А дальше через эту точку проводят нагрузочную прямую с таким наклоном, чтобы верхний отрезок(до нуля на сетке) получился в 1,1...1,2 раза длиннее нижнего отрезка(до удвоенного смещения). По этому наклону определяем Ra(приведённое сопротивление). Это по-простому. ... !!! Это справедливо для пентода/тетрода.

Пермяк: Попробовал - этот режим никак не подойдёт. 6п13с в пентоде - явно не для звука. Для импульсных режимов. В крайнем случае - для двухтакта в режиме класса Б.

B_Aleko: Пермяк пишет: Попробовал - этот режим никак не подойдёт. Спасибо. но ведь ее используют, но видимо в триоде? и тогда как? продолжить график, что не вмещается? может на этом примере можете показать: http://shot.qip.ru/00Qk6h-5d3R3GHLH/ по отзывам хороша, да и первое собирать, лучше, чем 6п14п.

Пермяк: Лампа 6п13с создана в начале 50-х гг. для работы в вых. каскаде строчной развёртки чёрно-белых телевизоров. Там она работала в пентодном включении. Для чего создана - для того и ВАХи сняты и приведены в справочниках. Других ВАХ производитель не дал. А вскоре были созданы и другие, более мощные лампы для такого применения. B_Aleko пишет: но ведь ее используют, но видимо в триоде?Да, исключительно в триоде. Те, кто пишет, что она якобы в пентоде хорошо "звучит", практически никогда не выкладывают данных о её режиме и схемном включении, поэтому заявления их сомнительны. И ещё. Применение пентодов и тетродов усложнено ещё и тем, что часто экранная сетка должна питаться от значительно меньшего напряжения, чем анодное. Обратите внимание на подпись под картинками ВАХ 6п13с и 6п15п от Бройде. Посмотрите, как влияет напряжение экр. сетки Uc2 пентода на его ВАХи. Есть, правда, некоторые методы "исправления" пентодных характеристик в динамическом режиме с помощью местных обратных связей. Два таких метода рассматривались на форуме в разделе Теория. http://hiend.borda.ru/?1-13-0-00000036-000-0-0-1399180471 http://hiend.borda.ru/?1-13-0-00000038-000-0-0-1398743925 ................................... Недостающие ветви ВАХ на графиках можно дорисовывать. Очень часто приходится дорисовывать ветви промежуточные, если РТ оказывается между имеющимися на графике ветвями. При некотором навыке можно на триодном графике дорисовывать ветви справа, если уловить закономерность уменьшения расстояний между ветвями. Линии ВАХ пентодов можно продолжать вправо, т.к. в зоне больших анодных напряжений они обычно близки к прямым линиям.

aur_100: Не понимаю, зачем хотят пентодный(тетродный) звук от телевизионных ламп. Триодный, достаточно большая мощность, низкие искажения...

andreii: Пентодный режим вообще не для звука, одни искажения, визг и хрип, если нужен итун то нужно делать выходной каскад на триоде с общей сеткой, и раскачивать понижающим трансформатором через катод.

B_Aleko: Пермяк Вот спасибо. Очень познавательно. Со временем многое пойму. Так как за неимением графиков пентодов в триодном режиме нет, то и изучаю изначальное предназначение лампы. Сейчас пока передо мной один вопрос, о положении РТ выходных ламп, дорисовки графика, расчет анодного и внутреннего сопротивления (в этом вся загвоздка). потом расчет транса. aur_100 пишет: Не понимаю, зачем хотят пентодный (тетродный) звук от телевизионных ламп. Триодный, достаточно большая мощность, низкие искажения... сейчас просто изучаю теорию расчёта пентода. И что можно получить с лампы при использовании в разных режимах. По лампе 6н3п. Выбрал три режима с токами покоя: 7мА, сетка 2 В, 5мА, сетка 2.1В, 4мА, сетка 2В. По расчетам сопротивлений, токов и напряжений после сборки всех трех вариантов. получил расчетные режимы. Есть вопрос: для качества звучания какой наиболее предпочтителен режим: с меньшим током и большим анодным сопротивлением или большим током и меньшим сопротивлением?

aur_100: B_Aleko пишет: Выбрал три режима с токами покоя: По мне, 7мА, а еще лучше 2,5в 9мА, по мах...

B_Aleko: aur_100 чем это лучше? и как это сказывается на качестве?

aur_100: КНИ меньше, качество лучше, но Ку чуть меньше...

Stan Marsh: B_Aleko пишет: что можно получить с лампы Берём что-нибудь вроде 6П14П, характеристик с нагрузочными линиями в инете полно. И вникаем...вникаем. Ещё раз: каскад в резистором и каскад с трансформатором - это разные каскады; триод и пентод - это разные лампы.

B_Aleko: Резистивный каскад - понятен. Разобрался, смакетировал. Пока только на 6н3п, дальше другие на очереди. Все записываю на будущее. Вот с выходными... Понятно, что транс не может быть резистором, что бы обеспечить РТ с большим падением напряжения питания на нём, потому и РТ выбирают практически анодное. Принцип дорисовки графика до нужного напряжения из которого вычисляется анодное сопротивление - почти понятно. Как вести расчет? Аналогично резистивному или есть нюансы? При расчете брать полный размах при выбранном смещении? Принцип расчета нужен и для триодного, лучевого тетрода и пентодного включения.

B_Aleko: остается открытым вопрос: По лампе 6н3п. Выбрал три режима с токами покоя: 7мА, сетка 2 В, 5мА, сетка 2.1В, 4мА, сетка 2В. По расчетам сопротивлений, токов и напряжений после сборки всех трех вариантов. получил расчетные режимы. Сам вопрос: для качества звучания какой наиболее предпочтителен режим: с меньшим током и большим анодным сопротивлением или большим током и меньшим сопротивлением?

Пермяк: Вариант 7 мА - предпочтительней. Чем выше расположена РТ, тем меньше Ri, больше S. Это благоприятно влияет на качество звучания. БОльший анодный ток покоя - тоже само по себе неплохо. Вам об этом уже писали дважды в этой теме.

B_Aleko: вот теперь ясно. горизонтальную линию нагрузки сделать не реально, но расположить на максимально возможный угол с учетом мощности лампы, допустимых напряжений и токов.

ALSS: B_Aleko пишет: горизонтальную линию нагрузки сделать не реально Да запросто - генератор тока на транзисторах - еще 50-100 В или на лампах - еще 100-200 В добавить к питанию; посмотрите схемы Евгения Карпова из Одессы, например, его статью http://next-tube.com/articles/drv/drv_3.pdf.

B_Aleko: ALSS пишет: Да запросто - генератор тока на транзисторах это пока сложно для меня.

B_Aleko: есть небольшие сомнения. как высчитать такие параметры как: 1. ...Мощность рассеяния на аноде можно определить по формуле: Pa=Ua0*Ia0 или как ранее писали здесь: 0,9*Pmax/Ea? 2. ...Желательно также проверить тепловые режимы резисторов каскада. Мощность рассеяния на анодном резисторе составляет: PRa=IaO(Uao-UaO) (тут вроде ошибка?), или же по Ому?

B_Aleko: влияют ли на сопротивление катода Ra, Ri, Rc, если да то как? по расчету: Rk=Ug/Io. встречал дополнительные данные. нашел такое решение расчета катодного резистора от компании "Миллард": Rk=Rk+(1/S)

B_Aleko: нашел вот такой график расчета входного сеточного сопротивления с сайта: http://tubeamplifier.narod.ru/mess073.htm (Рис. 3.10 Предельное значение сопротивления в цепи сетки (по данным компании Миллард)) только не пойму как им пользоваться

ALSS: B_Aleko пишет: расчета входного сеточного сопротивления Обратите внимание, что это расчет предельно допустимого максимального значения сопротивления утечки сетки в статическом режиме - это может пригодиться для расчета электрометрической схемы, например. Для наших целей желательно, чтобы это сопротивление не слишком шунтировало источник сигнала или выходное сопротивление предыдущего каскада, чтобы не было их перегрузки и падения сигнала. Вполне достаточное для практики соотношение от 4 до 10 раз для минимального значения сопротивления утечки сетки (название исторически сложившееся, придираться не следует). Чем больше импеданс цепи, тем больше она подвержена действию наводок со стороны окружающей электромагнитной среды. И даже не надо ПСН сюда привлекать.

B_Aleko: ALSS пишет: чтобы это сопротивление не слишком шунтировало источник сигнала или выходное сопротивление предыдущего каскада но ведь должна же быть формула? а на "ощупь", как-то не хочется.

Пермяк: B_Aleko, формула есть, но практика показала, что для разных экземпляров ламп одного и того же типа величина тока сеточной "утечки" имеет довольно значительный разброс, буквально - в разы, причём ток этот увеличивается при длительной эксплуатации лампы. Практически же установлено, что для маломощных ламп достаточно ставить Rg=0,5÷1МОм. Чем меньше Rg, тем легче лампе, но хуже для предыдущего каскада или источника сигнала. Для мощных выходных ламп величина Rg обязательно оговаривается в даташите, причём для применения при фиксированном смещении его величина меньше, чем при автосмещении. B_Aleko пишет: есть небольшие сомнения. как высчитать такие параметры как: 1. ...Мощность рассеяния на аноде можно определить по формуле: Pa=UaO*IaO или как ранее писали здесь: 0,9*Pmax/Ea ? 2. ...Желательно также проверить тепловые режимы резисторов каскада. Мощность рассеяния на анодном резисторе составляет: PRa=IaO(Uao-UaO) (тут вроде ошибка?), или же по Ому? На п.1. Первая формула - определение значения рассеиваемой мощности на аноде при выбранных Uao и Iao. Вторая формула- для определения допустимого тока покоя по известному (по паспорту) значению Pmax и напряжению на аноде в выбранной рабочей точке. На п.2 Правильно будет так: PRa=Iao*URa, или: PRa=(Iao)2*Rа, или: PRa=Iаo*(Uпит.-Ua).

B_Aleko: Всем доброго дня! Делаю первый отчет. При сборке макета, из наличия 31 шт. ламп 6н3п, только одна оказалась в почти расчетном режиме (должно быть 125В, в итоге - 132В анодного и 2В на катоде (при питании 305В). Остальные стабильно работали при 141В анодного и 2В катода. Причём разброс режимов не превышал 0,05В в катоде и 140-142В в аноде. Далее, для проверки качества звучания, собрал простой каскад на 6п14п. Транс с активным сопротивлением в 190 Ом (его параметры, шл-0,052(50Вт)?, при подаче 220В, на выходе две обмотки по 8В), другого пока нет. Одну подключил к колонке, вторую - в цепь катода 6п14п. Использовать пробовал ТВЗ, но звук не понравился, поставил как дроссель. На очереди мотаю ШЛ 0,1(100Вт), с первичной в 3600 вит. с расчетами трансов пока не разобрался, но все впереди! Замерял тремя приборами: АВО63, Ц20 и цифровым M890D. в катодах резисторы С5-5 1%, в аноде, сетках - МЛТ 5%. Колонки не ахти, SVEN, с кевларовым диффузором 130мм, объем чуть более 12 литров. они никогда так не звучали! чисто, звонко. ставил на прослушивание много разных записей, голосовые партии прослушиваются с дыханием певцов проверить спектры, искажения не предоставляется возможным из-за отсутствия необходимых приборов.

B_Aleko: вот фото макета. с учетом, что собрано "на коленке" на деревянной основе, при максимальной мощности, фона и посторонних звуков абсолютно не слышно. единственное, это бывают совсем небольшие наводки от сотового. звучание по мощности сравнимо с моей акустикой в 35Вт, то есть приблизительно в 15-20Вт. (на слух).

B_Aleko: несколько замечаний о своих ошибках: 1. анодное измерял относительно общего. а между анодом и катодом, абсолютно согласно графическому вычислению = 125-126В! (соответствие по графикам Бройде). 2. формула Rk=Ugo/Igo+(1/S)=500+0.243=(округленно) 500,25 Ом!, что достаточно не ощутимо. 3. так как с выходными каскадами еще не совсем разобрался, возможно и там есть критические ошибки. смущает смещение в 8В, даже при отсутствии дополнительной обмотки. питание 305В, между анодом и катодом 260В. перепробовал 6 ламп, результат один.

Пермяк: B_Aleko пишет: перепробовал 6 ламп, результат один. А это в большой степени потому, что при автосмещении резистор Rк является также элементом стабилизации тока через лампу, и при этом устраняется некоторый разброс параметров разных экземпляров ламп.смущает смещение в 8В Следует помнить, что важно не напряжение смещения, а ток через лампу. Ток через Rк (и через лампу) равен Ia+Ic2, и этот суммарный ток у Вас равен 8В/130ом=0,062А=62 мА. Максимально допустимая мощность, рассеиваемая анодом и второй сеткой: 12+2=14 Вт При токе 62 мА и напряжении Ua-к=260 В рассеиваемая мощность будет равна: 260*0,062=16 Вт, что превышает допустимую. Надо подобрать Rк так, чтобы ток через него не превышал 14/260=54 мА. Но если при 62 мА анод лампы не перегревается (посмотреть в темноте - не краснеет ли анод или 2-я сетка), то можно оставить и так. Есть любители несколько перегревать ("жарить") лампы, при этом отмечают повышение качества звука. Не знаю, относится ли это к 6п14п , не такая уж она могучая...

B_Aleko: Пермяк пишет: чтобы ток через него не превышал 14/260=54 мА. да, так оно есть. поставил, проверил. Ua-k=270, Rk=110, I=0.054. Вот только никак не пойму. транс 190 Ом. как высчитать Ra?

Пермяк: B_Aleko пишет: как высчитать Ra? Вам нужно высчитать, какое Ra может дать трансформатор? Тогда сообщите: сопротивление Вашей АC, количество витков первичной обмотки и количество витков вторичной обмотки (которая подключена к динамику).

B_Aleko: увы, количество витков в трансе неизвестно. ас 4 Ом. сам хочу рассчитать на ОСМ 0,1, но пока не разобрался.

B_Aleko: по лампе 6Н3П по схеме Ri=(160-90)/(0.0096-0.0014)=8540 m=70/2=35 S=8.2/2=4.1 проверяем соотношение, должно быть равно 1: (S*Ri)/m=1.0004. Ku=(153-97)/2=28, или Ku=m*(Ra/(Ra+Ri))=29 как вычислить КНИ?

Пермяк: B_Aleko пишет: должно быть равно 1 Ещё бы ему не быть равным единице, если одна величина найдена из двух других! Таким способом можно проверить только не допущена ли ошибка в делении-умножении. Объективная проверка будет только в том случае, если Ri будет найдено независимо от двух других параметров. как вычислить КНИ? Это довольно сложно расписывать, особенно, если задействованы только две-три ВАХи... Дам ссылку на книгу. Вот библиотека со свободным скачиваанием: http://publ.lib.ru/ARCHIVES/_CLASSES/TEH_RAD/_Teh_rad.html Находите в списке авторов Цыкин Г.С. Щёлкаете по фамилии, откроется список его работ. Из списка выбираете Электронные усилители. 1965 г. (в красной обложке). Открываете стр.72 и далее.

B_Aleko: Пермяк пишет: Открываете стр.72 и далее. скачал. спасибо

B_Aleko: может зря спрашиваю: еще вопрос. если у 6п14п смещение 6В, а усиление 6н3п =28. амплитуда 6н3п =28, а входная 6п14п 12В? собрал выход на 6п13с, смещение -30В. звук громче, чище, но нейтральный. входная амплитуда в 30В? что то запутался...

ALSS: B_Aleko пишет: еще вопрос. если у 6п14п смещение 6В, а усиление 6н3п =28. амплитуда 6н3п =28, а входная 6п14п 12В? При смещении 8 В максимальный размах сигнала на сетке 6П14П может быть - чтобы не заходить в сеточные токи - 8х2=16 В. 16 В делим на коэффициент усиления предыдущего каскада 28 и получаем размах входного для 6Н3П сигнала 16/28=0,57 В (т. е. 0,57/2,82=0,2 В скз). Почему же Вы вставляете 1 В и дальше носитесь с получающимися 28 В?! Не подавайте 1 В и не будет 28 В на аноде 6Н3П.

Пермяк: B_Aleko Можно попробовать убрать конденсатор, шунтирующий Rк лампы 6п14п. Её чувствительность уменьшится. Также можно попробовать убрать такой же конденсатор из катода 6н3п. Ку драйвера тоже снизится, хотя и немного, думаю. Оба варианта тщательно отслушать, поочерёдно и совместно. B_Aleko пишет: собрал выход на 6п13с, смещение -30В. звук громче, чище, но нейтральный. входная амплитуда в 30В? Вот это - правильно. Триод в вых. каскаде - много лучше, чем пентод. Амплитуда сигнала на сетку 6п13с поступает, возможно, и меньше, чем 30 В, зато не вгоняет её в положительные значения на сетке. Отсюда и лучшее звучание. И выходной трансформатор для триода изготовить проще. Если Вы не знаете моточных данных выходного транса, то хотя бы измерьте приблизительно его коэффициент трансформации Ктр. Подайте на первичку напряжение U1=220В (а если есть ЛАТР, то вольт 100), и измерьте напряжение U2 на вторичной обмотке, которую будете потом подключать к динамику. Ктр=U1/U2

B_Aleko: пока думаю, чем раскачать 6П13С. смотрю графики триодов, способных дать усиление более 28. вместо 6П14П, поставил 6П18П со смещением в 10В. если не делать полную громкость, звучит ярче, детальней, чем 6П14П. но звук действительно пентодный... не сравнимо конечно. он жестче...

ALSS: Поставьте 6П43П со смещением 25 В. В триоде, конечно. А зачем Вам триод с усилением более 28-ми? Ведь и так чувствительность получается 0,2 В скз, любой фонокорректор можно ко входу подключать, а сигнал от проигрывателя компакт-дисков придётся давить в 10 раз, что приводит к завалу ВЧ из-за фильтра из сопротивления регулятора уровня и входной емкости (монтажной и Миллера) каскада на 6Н3П. 6Ж52П в триоде имеет мю под 80, и 6Ж51П в триоде мало чем уступает, если речь идет о пальчиковых лампах. Октальные - и 6Н9С, и 6Г2, и 6Г7. Кстати, прекрасная связка на 6Г2 и 6П13С (Манаков? Сергеев?) неоднократно с успехом повторена http://rh.adsh.org.ua/search.php?search_id=1404827866

омельян: http://rh.adsh.org.ua/viewtopic.php?id=2472 Усилитель собран, работает прекрасно. Режимы указаны на схеме.

B_Aleko: ALSS пишет: 6Ж52П в триоде имеет мю под 80, и 6Ж51П в триоде 6ж52 есть несколько, но все б/у. 51-я - только одна. тренировался с 6ж9п, пока не получается рассчитывать по пентодным графикам. должно быть усиление более 45, а в итоге работает тише, чем триод. А режим 6н3п менять не хочется (приберегу куда-нибудь), уж сильно точно встали параметры и по графическим расчетам КНИ =0,02% вот 6г2 и 6н9 - собрал бы, но нет таких. Пермяк пишет: измерьте приблизительно его коэффициент трансформации Ктр спасибо. так и сделаю.

B_Aleko: ALSS пишет: Поставьте 6П43П со смещением 25 В. В триоде Подумаю. График ВАХ нашёл. Но в толк не возьму: для 14-й питание 300, ток 45-50мА, см -6-8В. А на графике 43-й, при -25В - максимально 235В питание. Это уменьшать питание?

Пермяк: B_Aleko пишет: Это уменьшать питание?Конечно. Если Вы подбираете к драйверу выходной каскад (да ещё - по напряжению его смещения!), т.е. решаете задачу наоборот, то и источник питания придётся менять, да и выходной транс, скорее всего -тоже.

aur_100: Поставьте драйвером к 6п13с лампу 6н7с, лучше "Фотон", забудете другие...

B_Aleko: Пермяк пишет: Если Вы не знаете моточных данных выходного транса нашел данные транса в нете: Трансформатор Т2.2025.06 (220В n=1500 d=0.2mm,) ((16.7+/-0.8В)x2, n=230.d=0.41mm ) (22-23=6.5+/-0.3В, n=45 d=0.33mm)

омельян: http://rh.adsh.org.ua/viewtopic.php?id=2472&p=2 Очень жаль, что у Вас нет 6Г2. Звучание порадовало бы Вас. Я пробовал 6Ж4 и 6Ж8, но звучание было хуже. Пентоды не понравились. Даже в триоде. Вы сравните ВАХ 6Г2 и 6Н3П. Какая из них более линейная.

B_Aleko: омельян пишет: Очень жаль, что у Вас нет 6Г2.интересно. Она на "Истоке" есть, надо бы заказать. Дети идею корпуса усилителя и колонок подкинули. даже не ожидал. а под них нужны лампы такие на вид "старинные". вот теперь на выход надо что то фигурное. только сначала надо разобраться с расчетом трансформатора и подкрепить знания в расчете выходных ламп.

aur_100: Если ставить с высоким мю, то 6г7, усиление поменьше, но зато Ri меньше, усиления с 6п13с хватает, можно пробовать отключать Ск.

омельян: B_Aleko пишет: надо разобраться с расчетом трансформатора... Если хотите, я могу дать Вам расчёт вых. трансформатора на ОСМ-0,16. Получается очень хорошо. Вых.мощность 4 Вт. Этого хватает "с головой" для домашнего прослушивания.

B_Aleko: омельян пишет: Если хотите, я могу дать Вам расчёт вых. трансформатора вот это интересно. на этой основе можно ведь разобраться как рассчитывать?

омельян: Основу я Вам уже давал - книга Кризе. Дайте И-мейл. Отправлю Вам книги. См. ЛС.

B_Aleko: misteraleko@gmail.com

Пермяк: B_Aleko, В той книге Г.С. Цыкина, что Вы уже скачали, есть раздел по расчёту выходных трансформаторов. Кризе у хайэндщиков давно не котируется. B_Aleko пишет: нашел данные транса в нете: Трансформатор Т2.2025.06 (220В n=1500 d=0.2mm,) (16.7+/-0.8В)x2, n=230.d=0.41mm ) (22-23=6.5+/-0.3В, n=45 d=0.33mm)Это что, Вы в своём макете применяете в качестве выходника сетевой понижающий транс 220в ? Без немагнитного зазора?

омельян: Пермяк пишет: Кризе у хайэндщиков давно не котируется. Да, согласен. Но в данном случае - это очень даже подойдёт. Практикой подтверждено.

B_Aleko: Пермяк пишет: Это что, Вы в своём макете применяете в качестве выходника сетевой понижающий транс 220в ? Без немагнитного зазора? к сожалению..., но ведь работает уже три дня, не греется, лампы в норме.

Пермяк: А с чего бы ему греться-то? Поверьте, "не греется" - не самый положительный параметр выходника. В, общем, раз коллега омельян за Вас взялся - удачи, и Кризе - Вам в помощь!

омельян: Пожалуйста, намотайте вых. трансформатор по правильным расчётам и выберите правильные режимы. Ведь иначе звук будет непредсказуемым, что подрывает основы высококачественного звучания ламповой аппаратуры. Не помню, кто сказал: "Нужно делать хорошо, плохо у нас и так получается". to Пермяк: Я просто подумал, что на данном этапе Кризе вполне достаточно. Цыкин - корифей.

B_Aleko: омельян пишет: "Нужно делать хорошо, плохо у нас и так получается"надо делать плохо, у нас и так хорошо получается. - антоним. Вообще-то, благодаря данному форуму, многое изучил, есть книги, формулы... на 6п13С, откладываю в долгий ящик. не нравится. А вообще-то, 6н3п очень хороша. звучит прекрасно при разных режимах, главное - подобрать точку РТ по минимуму искажений. В запасе еще есть 3 шт. 6п6с (б/у), 6 шт. 6п3с (б/у), 5 шт. - 1540 (вроде аналог 6п3с, б/у), 6п31С- 10 шт. (б/у) - только вряд ли с них будет толк. Зато хорошо потренируюсь в графиках и вычислениях. 6с33с - 4шт. (прямо с завода парные. давно очень заказывал). на них вряд ли скоро что то сделаю... Надо не только для себя стараться, а стараться для семьи. Привнося удовольствие стремлением свой окружающий мир сделать ярким и незабываемым!

омельян: B_Aleko пишет: на 6п13С, откладываю в долгий ящик. не нравится. А вот это зря. Лампа очень линейная. В триоде. Научитесь смотреть на ВАХ ламп.

aur_100: С сетевым трансформатором вместо выходного делайте сами из чего хотите и как хотите...

юрий 1958: омельян для прослушки симф. музыки может 4 вт. и хватит , но без барабанов , они там редкость .А вот чтото приличное послушать .... можно , но без объемных низов.Прослушайте последний альбом Гилмора ! Пока не навалил ватт 20 на щиты4а32 баса неастоящего не услышал ! Попробуйте !

ALSS: юрий 1958, у омельян есть его работы двухтактные моноблоки на 6С33С, он знаком с повышенной мощностью... омельян пишет: B_Aleko пишет: цитата: на 6п13С, откладываю в долгий ящик. не нравится. А вот это зря. Лампа очень линейная. В триоде. Научитесь смотреть на ВАХ ламп.Полностью поддерживаю. Все части данного сообщения от омельян. Ну, B_Aleko, из "маломощных" остается 6П41С в триоде, если крутиться около 4 Вт. Но использование сетевого трансформатора без зазора указывает на занятие процессом без представления о конечном результате, как это ни печально. В электрическом смысле. В музыкальном, т. е. для прослушивания музыки - знакомые музыканты слышат все что их интересует на любом звуковоспроизводящем агрегате (и в любом состоянии, но это уже другой вопрос). Если Вы обладаете такими способностями - я Вам завидую, мне же нужен хороший звуковоспроизводитель.

B_Aleko: я знаю, что много делаю ошибок. но без практики они - пшик. пусть пройдет время, в котором обрету истину, и возьму опыт людей.

Пермяк: О выходном трансформаторе. Очень хотелось бы прикинуть, а какую нагрузку Вы дали лампе посредством этого выходника, что получили от неё такой чудесный звук? Вы пишете:нашел данные транса в нете: Трансформатор Т2.2025.06 (220В n=1500 d=0.2mm,) ((16.7+/-0.8В)x2, n=230.d=0.41mm ) (22-23=6.5+/-0.3В, n=45 d=0.33mm), но при этом не сообщаете, которую из вторичек подключили к динамику, а которую - под катод? И какие размеры имеет сердечник этого трансформатора?

B_Aleko: Пермяк, извините, но я не писал, что звук чудесный, просто получил хорошее звучание. Для сравнения использовал исполнителей "Пентатоникс", у них приличные голосовые партии. https://music.yandex.ru/artist/1225394/similar а также середины прошлого века: Dee Clark По трансформатору: обмотки n=1500 d=0.2mm, - первичка n=230 d=0.41mm - на динамик n=45 d=0.33mm - катодная обмотка размеры сердечника у1=20, у2=25, b=10, h=40. слушаю на нем сейчас Bobby Bland, не супер конечно, но достойное звучание блюза. https://music.yandex.ru/artist/3099

B_Aleko: вопрос по ВАХам. к примеру есть 6п14п, у нее на графике пентодный режим при напряжении 2 сетки = 250В. режим ее работы установил в точности. а вот 6п18п нигде нет при таком напряжении сетки. как ее перерисовать (чтобы высчитать режим в пентодном включении. оч. хочу разобраться в пентодных режимах.)

Stan Marsh: B_Aleko пишет: 6п18п нигде нет при таком напряжении сетки. А она и не предназначена для "такого" напряжения. 170В - это её предел. Не считайте разработчиков ламп глупее себя, они дают оптимальные режимы. ... А для триодного включения картинок полно.

B_Aleko: Stan Marsh пишет: ...они дают оптимальные режимы. я и не считаю. но многие лампы не предназначены для звука и в разных режимах. ведь разработчики тоже под это их не разрабатывали...

Stan Marsh: B_Aleko пишет: многие лампы не предназначены для звука и в разных режимах. Про 99% ламп уже давно всё известно. Остальные неинтересны или просто недоставаемы. ...

ALSS: B_Aleko пишет: По трансформатору: обмотки n=1500 d=0.2mm, - первичка n=230 d=0.41mm - на динамик n=45 d=0.33mm - катодная обмотка размеры сердечника У1=20, у2=25, b=10, h=40. 4,5 кв. см сердечник (kст=0,9) - ватта на 2 хватит, ну, 3. Ктр=230/(1500+45)=0,15^2=0,022 4 Ом/0,022=180 Ом приведенного к аноду сопротивления нагрузки?! Издеваетесь над лампой (и не только над нею). Может, 230 витков в катодной обмотке (около 13% - допустимо), а 45 - на динамик? Тогда Ктр=45/(1500+230)=0,026^2=0,00068 и приведенная анодная нагрузка 4/0,00068=5,9 кОм - о, это уже похоже на необходимое лампе.

омельян: ALSS пишет: 4,5 кв. см сердечник (kст=0,9) - ватта на 2 хватит, ну, 3. Да, вроде как нашли консенсус. Нужен сердечник от ОСМ-0,16. Будем искать.

Пермяк: B_Aleko пишет: многие лампы не предназначены для звука и в разных режимах. ведь разработчики тоже под это их не разрабатывали... 6П18П - особая лампа. Она разработана так, что при относительно малых напряжениях (Uaо=Uс2о=150v) она способна давать относительно большой ток анода. В той же библиотеке http://publ.lib.ru/ARCHIVES/_CLASSES/TEH_RAD/_Teh_rad.html скачайте: Терещук Р.М. "Справочник...", 1962, и со стр. 402 и далее прочитайте §6 о схемах, в которых она могла применяться. ЗЫ. Нескромный вопрос: а осциллограф у Вас есть? Понятно, что конечным результатом является ЗВУК, но без осциллографа оценить работу усилителя (даже по расчётам), на мой взгляд - невозможно.

B_Aleko: Добрый день. Позже отпишусь по вопросам и вычислениям. Верстаю очередной номер журнала. Много работы. думаю на неделю. А по 6п18п, при питании 170-190 В ток покоя получается выше, а значит и подмагничивание транса будет больше.

B_Aleko: Stan Marsh пишет: А она и не предназначена для "такого" напряжения. выкраивал время, чтобы экспериментировать и отрисовывать график лампы (черно-серые линии соответствует графику Бройде). при увеличении напряжении второй сетки до 290 В, график строился таким образом, первый результат: при второй сетке в 190 В, второй результат: для расчета принимал различные амплитуды входного сигнала. при имеющемся трансформаторе с омическим сопротивлением первички в 99 Ом, получился результат, близкий второму варианту. вот график результата (синим указано использование транса): и последнее, есть транс ТВЗ 1-6, его сопротивление 274 Ом, и результат оказался значительно плачевный, и при 190 В, и при 290 В. звук - ужас.

B_Aleko: вот схема. немного изменена и с учетом выходного транса. собрана по синему графику ВАХ уменьшил усиление первой лампы. повторил схему на другом силовом, и выходном таком же трансе не подбирая лампы и детали, "на бум", результат оказался очень и очень хорош.

омельян: При таком раскладе у Вас вторая сетка работает как анод. Напряжение на 2 сетке не должно превышать напряжение на аноде.

B_Aleko: омельян пишет: При таком раскладе у Вас вторая сетка работает как Анод Отчего же она так работает? Параметры не превышены. Напряжение на 2 сетке не должно превышать напряжение на аноде. В литературе не встречал таких ограничений. Есть много примеров подключения второй сетки напрямую к питанию, в том числе - и в справочнике для 6п18п: Или что-то пропускаю? Два полных дня работает, можно на пару сек прикоснуться, жаром не пышет (намного меньше греется, чем 6п14п). миллиамперметр в катоде показывает стабильно 58-59 мА. у соседа усилитель pioneer a 50, так сравнили (при приблизительно одинаковой громкости). сосед будет продавать свой...(шутка, оч понравился звук, может закажет ламповичёк. но он любит большую громкость).

Stan Marsh: омельян пишет: Напряжение на 2 сетке не должно превышать напряжение на аноде. При типовой схеме включения выходного пентода напряжение второй сетки всегда больше напряжения анода на величину падения напряжения на активном сопротивлении первичной обмотки выходного трансформатора. В пределах 10В. Это нормально.

B_Aleko: По искажениям. на графике видно. нижний хвост красной линии совсем немного выступает за линию -13. так эту линейность (что бы был линейный участок), с помощью изменения тока покоя, можно сместить и вместо 60 мА, выставить 58 мА. тогда смещение будет не -7,00, а -7,05 - -7,1 В к примеру. и не подавать на вход лампы размах более 13 В. у меня чуть более 13 В. По блоку питания: Транс ТС-180, перемотанный кем-то, брал с рынка. При выключенных всех приборах (в общем - перед сном), шума и фона НЕТ!

омельян: Ну, вот и хорошо. Теперь выберите правильную ВАХ (рабочую точку) для драйверной лампы.

B_Aleko: омельян пишет: Ну, вот и хорошо. Теперь выберите правильную ВАХ (рабочую точку) для драйверной лампы. вот действительно, поспешишь... извиняюсь, подкорректировал график 6Н3П. у лампы 6Н3П хорошую линейность можно получить при смещении -2, и... при -4, практически абсолютная. Заметил одну деталь: драйвер лучше запитывать напряжением выше, чем выходной каскад. так как больше динамический диапазон и большая линейность.

Пермяк: B_Aleko пишет: есть транс ТВЗ 1-6, его сопротивление 274 Ом, и результат оказался значительно плачевный, и при 190 В, и при 290 В. звук - ужас.ТВЗ1-6 - выходник для ДВУХТАКТНЫХ схем вых. каскада, а значит - в сердечнике отсутствует немагнитный зазор, и при применении в однотакте его сердечник будет работать в области насыщения, то есть - никак. Приблизительный размер зазора lз определяют по формуле: Толщина немагнитной прокладки равна lз/2. Требуется сердечник ТВЗ1-6 перебрать "встык" и проложить немагнитную прокладку. На практике толщину прокладки уточняют подбором по максимальной индуктивности первичной обмотки. Вот данные ТВЗ1-6 и значения Ra, которые он может дать при разных нагрузках: http://shot.qip.ru/00Buq9-6IRG4yxUQ/

B_Aleko: Всем вечер добрый. Проверил в макете у лампы 6н3п ранее оговоренную РТ на ВАХ: смещение -4В, питание 275В, ток 3мА. результат реально ошеломил. Такая динамика, детальность. И почему эту лампу считают не подходящей? Она способна на большее, если внимательно к ней отнестись. Проверил все свои 6н3п в количестве 28шт. Есть новые и б/у. Результат оказался на 90% "в точку". При других режимах разброс составляет намного больше, совпадение уже менее 60%. (по графикам Бройде).

Пермяк: B_Aleko пишет: ранее оговоренную РТ на ВАХ: смещение -4В, питание 275В, ток 3мА Коллега, мой Вам совет: когда выкладываете данные РТ, то указывать надо напряжение анод-катод или просто напряжение на аноде, а не "питание". Обозначения лучше применять следующие: Ea - напряжение питания анодной цепи каскада; Ua0 (или Uao) - напряжение анод-катод в режиме покоя (в точке РТ); Ua - напряжение анод-катод в любой рассматриваемой точке. Тогда все будут понимать, о чём Вы пишете. -------------------------------- График из Бройде, который Вам всё время подсовывают коллеги :) , для оценки 6н3п плохо пригоден. Я давал Вам приличные, вполне информативные ВАХи от Клауса и Алекса, даю ещё одну: А по этому графику видно, например, что при смещении -4 вольта, Ri в 2 с лишним раза больше, и S в три раза меньше, чем при -2В.

aur_100: B_Aleko, лампа 6н3п - хуже некуда, ее ставят в УВЧ, гетеродины и пр., но никак не качественные УНЧ...

Сергеев Сергей: aur_100 пишет: лампа 6н3п - хуже не куда, ее ставят в УВЧ, гетеродины и пр., но никак не качественные УНЧ... Не согласен. И не только я, но многие, в том числе и Ю.Макаров.

aur_100: Сколько я ни пытался, все равно - "транзисторное" звучание, даже в корректоре с малыми сигналами, имхо...

B_Aleko: aur_100 пишет: Сколько я ни пытался, все равно - "транзисторное" звучание, даже в корректоре с малыми сигналами, имхо... здесь не согласен. Отличия ощутимы при сравнении с хорошим каменным. электролит в катоде не ставлю. максимум до 2 мкф пленочный.

Stan Marsh: B_Aleko пишет: 2 мкф Равносильно его отсутствию.

B_Aleko: Stan Marsh пишет: Равносильно его отсутствию. большинство в иностранных и старых схем вообще конд отсутствует.

Stan Marsh: Отсутствие конденсатора=ООС, малая ёмкость=частотно зависимая ООС, большая=отсутствие ООС.

B_Aleko: журнал сдал в печать. если кому интересно можете посмотреть: http://ru.calameo.com/read/003954547325294038531 я, дизайнер-верстальщик. если кому надо брошюры, буклеты и т.д., обращайтесь. мой сайт: http://misteraleko.wix.com/misteraleko По теме: пока не переберу режимы 6н3п, к другой лампе не перейду. И пусть я потрачу много времени на каждую, но знать буду, какая лампа может подойти в каждом конкретном случае, и как правильно выбрать её режим.

ALSS: B_Aleko пишет: Пока не переберу режимы 6н3п, к другой не перейду. Зачем тратить время на мельчайшие детали режимов?! Поверьте, Вы потом пожалеете о затраченном на ненужные подробности времени. Задачу надо решать крупными мазками, ну а потом шлифовать. Позволю привести собственный пример - если бы я влезал в тонкости движения электронов и абсолютно все влияющие факторы, я бы не разработал несколько сотен устройств от контактных групп до автоматизированных измерительных систем и в серийное производство пошли бы не десятки, а едва единицы изделий. Я понимаю, что Вы более чем на 20 лет младше по возрасту, но о времени - если оно ценно - задумываться надо было еще 20 лет назад. Простите за стариковское брюзжание, но слушайте музыку - если она Вам интересна - на сейчас работающей системе и улучшайте ее звучание, если чувствуете необходимость. Или повторите - не думая! как ни странно это звучит - удачную разработку и идите дальше, уже имея успешный опыт за плечами.

B_Aleko: ALSS пишет: ...идите дальше, уже имея успешный опыт за плечами время ценно, но без времени не будет опыта. спасибо.

B_Aleko: В цепи катода имеется некое сопротивление, которое создает местную ОС. его шунтируют емкостью. Вопрос: выбор емкости определяется частотой среза по НЧ? или чтобы полностью исключить ОС? При использовании формулы Xc=1/(2п*f*C), выбрал нижнюю частоту 35-40 Гц, емкость составила 4,4 - 4,7 мкф, и реактивное сопротивление емкости 982 Ом. Так как сейчас использую лампу с катодным сопротивлением в 1 кОм, возможно ли, что они компенсируют как то друг друга, имея практически одинаковое сопротивление? Резистор по постоянному напряжению, а конденсатор по переменному. какой будет коэффициент местной ОС?

ALSS: 1. Все классические расчеты приведены по уровню напряжения минус 3 дБ (0,707), что в конечном итоге дает уменьшение мощности в два раза. Этого достаточно для озвучивания, но не для воспроизведения музыки, для воспроизведения музыки желательно иметь падение мощности не более 0,5-1 дБ на заданной - нижней или верхней - частоте. 2. Одним из критериев, ранее (до второй мировой) никак не рассматривавшихся в быту, является фазовый сдвиг на НЧ. Приемлемым считается значение не более 10 градусов на 40 Гц, что приводит к нижней на уровне минус 3 дБ 40/(2pi)=6,3 Гц - вот на этой частоте реактивное сопротивление конденсатора катодной цепи не должно превышать 10% от сопротивления катодного резистора, что для 1 кОм дает 100 Ом и 250 мкФ (перепроверьте, пожалуйста, считал на ходу к койке). 3. Гораздо более жесткое требование выдвигает Макаров Ю. А. - это спад импульса длительностью 1 с не более 0,17 дБ и отсюда С=50хI/U, что для 1 кОм и 10 мА требует 50*0,01/10=0,05 Ф (50000 мкФ). В реальной практике достаточно одной десятой этой величины. 4. Все вышеприведенное является не более чем моим вкусовым предпочтением. И мне ни разу не было плохо. Мало того, позволяло избавиться от "сверчково-цикадного" звучания юго-восточных усилителей любителям европейской классической музыки и всемирного рока. 5. А считать коэффициенты обратной связи? Ее не должно быть. Ну разве что местная в выходном каскаде.

B_Aleko: ALSS пишет: ...Приемлемым считается значение не более 10 градусов на 40 Гц... Как это высчитывается? Для меня пока это темный лес. ограничение до 10% что бы завал был на 3дб? а если больше ограничить нижнюю частоту. извиняюсь за пока некомпетентность в данном вопросе. Почти в точку. пересчитал. можно 220+47 мкф при 99.35 Ом. 6,3 Гц, 265 мкф, 100.097 Ом.

Пермяк: B_Aleko пишет: Как это высчитывается? Но Вы ведь уже рассчитали минимально необходимую величину Ск. Поясню. По формуле, выложенной Александром, Вы определили Ск при условии, что на частоте 6,3 Гц синусоидальный сигнал уменьшится до 0,707 от уровня на средних частотах, т.е. уменьшится в 1/0,707=1,41 раз. Это в децибелах составит спад уровня на 3 дБ. При такой величине Ск, если подать на вход каскада сигнал типа меандр частотой 40 Гц, то спад вершин прямоугольных импульсов меандра не превысит 10%. Ну, есть такая найденная эмпирически норма, определённая "на слух", с которой, кстати, далеко не все хайэндщики согласны. Оговорюсь, что формула эта несколько приблизительна, но вполне достаточна для практических целей. Классический расчёт величины Ск сложен, особенно для пентода. Я мог бы предложить Вам ознакомиться с ним в имеющейся у Вас "красной" книге Г.С.Цыкина (стр. 304-307 и далее), но не буду этого делать :). Теперь - о главном. В усилителе, как правило, несколько каскадов, минимум 2. И если каждый каскад будет давать спад 3дБ, то на выходе усилителя спады эти просуммируются, и дадут недопустимый для нас общий спад уровня на НЧ. По этой причине (да ещё из-за трудностей с точными книжными расчётами), на практике, величину Cк для предватительных каскадов выбирают много больше расчётной, в 10-20 и более раз. Да и для вых. каскада - тоже. И долго спорят, какой фирмы-изготовителя следует покупать эти конденсаторы :). Частенько нелюбовь к конденсаторам народ преодолевает разными неклассическими решениями: ставят в катод батарейку, либо стабилитрон, либо светодиод, и т.д. Но это - совсем другая история...

B_Aleko: Пермяк пишет: на частоте 6,3 Гц синусоидальный сигнал уменьшится на 3 дБ. хорошо, но если мне нужен спад ниже 40 Гц? для уменьшения габаритов выходного транса.

ALSS: В случае сужения полосы сигнала снизу для уменьшения габаритов выходного трансформатора просто подставляется другая частота в расчеты и обязательно вводится ограничение полосы перед/между каскадами, чтобы на этот уменьшенный трансформатор не пришли частоты ниже его рабочего диапазона и не ввели его сердечник в насыщение. Таким образом пытаются делать усилители для мидбаса или для ШП с поддержкой снизу. Ну и ПРО-системы озвучивания. Вопросы задержек/опережения сигнала в полосах трогать не будем. Конечно же следует учитывать эффект накопления ограничений, о котором пишет Пермяк, вследствие увеличения порядка фильтра. Для озвучивания этот эффект полезен. Однако тут вступает в действие субъективный эффект - чем сильнее ограничена полоса усилителя снизу, тем мельче масштаб воспроизводимой музыки. Для музыки лучше ограничивать - по напряжению! Не по полосе - входной сигнал и работать на меньшей мощности, но в максимально широкой полосе. О том, что это мое субъективное мнение, напоминать далее не стОит? Пермяк пишет: меандр частотой 40 Гц, то спад вершин прямоугольных импульсов меандра не превысит 10% Нет, к сожалению, для 10% спада вершины на частоте меандра 40 Гц нижняя частота д. б. гораздо ниже, 0,16*0,1/0,0125=1,28 Гц. Впрочем, в данном случае это мелочи второго порядка малости.

Пермяк: Да, прошу прощения, ошибся: при Fн(-3дБ)=6,3 Гц скол импульса меандра частотой 40 Гц равен 50%.

Пермяк: B_Aleko пишет: мне нужен спад ниже 40 Гц... для уменьшения габаритов выходного транса.Коллега, если бы Вы сразу задали вопрос именно так, то сразу получили бы конкретный ответ: для этой цели Ск как правило не применяется. Предположим, что мы постепенно уменьшаем частоту синус-сигнала от СЧ до НЧ, и следим за уровнем на выходе каскада. Можно подобрать ёмкость Ск так, что на какой-то частоте, скажем 30 Гц, спад будет равен 3дБ. Но если мы и дальше будем уменьшать частоту, то с некоторого момента увидим, что спад постепенно уменьшается, и на какой-то частоте спада вообще не будет. Просто потому, что на этой частоте ёмкостное сопротивление конденсатора Ск уже очень большое, он практически не шунтирует Rк, и усиление сигнала каскадом (хотя и охваченным ООС) сохраняется вплоть до 0 Гц. Примеры вида АЧХ на НЧ при наличии Ск. Начало спада и его прекращение зависят от исходных параметров лампы и соотношения величин Rа и Rк, и зависимость эта достаточно сложная. Вывод: ожидать от Ск эффективной работы в качестве фильтра не следует.

B_Aleko: Пермяк пишет: получили бы конкретный ответ: как-то в начале не подумал о трансе. тогда: при спаде усиления, к примеру в 3 дб., в драйвере, можно настроить последующую цепочку из Rc и Cp тоже со спадом аналогичной частоты. и получим уже в 6 дб.?

B_Aleko: при Rc=220 кОм и Cp=22нФ, спад при частоте 32,88 Гц. если правильно, тогда на сколько дб?

Пермяк: На пост 165: На этой частоте суммарный спад, обусловленный этими двумя цепочками, будет 6 дБ. Но на более низких частотах крутизна спада уменьшится.

B_Aleko: то есть, уровень на частоте снизится и останется без изменений вплоть до 0Гц?

Пермяк: Нет. Цепочка CрRc по-прежнему будет давать спад, и при 0 Гц (постоянный ток!) не пропустит ничего :) B_Aleko пишет: при Rc=220 кОм и Cp=22нФ, спад при частоте 32,88 Гц. если правильно, тогда на сколько дб?Если имеете в виду только эту цепочку, то о ней всё сказано здесь: http://hiend.borda.ru/?1-13-0-00000093-000-0-0-1453553729

B_Aleko: Добрый день. У меня возникло сомнение о величине: Внутреннее сопротивление Ri - сопротивление лампы переменному току. Определяется как отношение изменения анодного напряжения к изменению анодного тока при неизменных напряжениях на остальных электродах. Измеряется в кОм. Внутреннее сопротивление может быть определено по значениям крутизны характеристики S и коэффициента усиления M: Ri=M/S Тогда чем отличается сопротивление постоянному току от сопротивления по переменному току?

Пермяк: B_Aleko Здравствуйте! Вы задали очень интересный вопрос. Допускаю, что с математикой у Вас не очень, поэтому попробую на этот теоретический вопрос ответить доступным (как мне кажется) языком. Во-первых, неверны сами по себе выложенные Вами определения. Вот как надо: 1. Сопротивление лампы постоянному току - это сопротивление в заданной (или выбранной) точке на семействе статических! ВАХ лампы. Определяется по закону Ома для пост. тока для неизменных Ua и Ia .Этому сопротивлению даже буквы специальной не присвоено, настолько оно не нужно при расчётах. 2. Ri - это сопротивление, проявлямое лампой в динамическом режиме, т.е. - при подключенной к ней нагрузке и изменении Ua и Ia при движении рабочей точки по нагрузочной линии (в нашем случае - по прямой Ra). Образно можно описать процесс так. Под воздействием изменения Uc рабочая точка пересекает линии ВАХ, "наталкиваясь на них поочерёдно. При этом ей кажется, что она движется по плоскости, сплошь заполненной бесконечным множеством линий ВАХ, которые имеют примерно одинаковый наклон. И переменный ток через лампу "видит" сопротивление, которое равно dU/dI. Что такое это самое dU/dI, Вы уже знаете: оно и есть Ri. Если моё пояснение что-то просветило... А если нет - то придётся углубляться в математику. Посмотрим.

Пермяк: to B_Aleko Я удалил из темы весь ранее следовавший за предыдущим постом флейм. Вы проводите какие-то исследования, касающиеся параметров лампы? Хорошо. Когда закончите эту работу, милости просим сюда, выложить результаты. Желательно на понятном русском языке, чтобы не пришлось Вас переспрашивать.

B_Aleko: хорошо. согласен. Только моя просьба остается в силе. конфиденциально, в личку контакты.

B_Aleko: Хотелось бы вернуться к вопросу о величине Ri. Это же тема о теории. Пермяк пишет:1. Сопротивление лампы постоянному току - это сопротивление в заданной (или выбранной) точке на семействе статических! ВАХ лампы.Это то же сопротивление лампы? но как быть с физикой? ведь не существует разных сопротивлений для двух величин (под переменный и постоянный ток). тем более в одном баллоне.

Ученик: B_Aleko, Вам уже ответил Пермяк:Этому сопротивлению даже буквы специальной не присвоено, настолько оно не нужно при расчётах. B_Aleko пишет: но как быть с физикой? Учить.ведь не существует разных сопротивлений для двух величин (под переменный и постоянный ток). Ещё как существует.

Пермяк: Попытаюсь ещё раз. Подключим к выводам накала лампы полагающееся ей по паспорту напряжение. Подключим между сеткой и катодом лампы источник напряжения 3В "минусом" к сетке. Между катодом и анодом подключим регулируемый источник напряжения "минусом" к катоду, и установим на его выходе напряжение 150 вольт. Согласно графику семейства ВАХ ток через ламу будет равен 5 мА. Если взять отношение абсолютных значений напряжения и тока, получим сопротивление пост. току: R=U/I=150/5=30 кОм. Проведём прямую АВ. Отметим, что для любой точки на этой прямой вычисленное R всегда будет равно 30 кОм. Т.е., получили сопротивление лампы постоянному току. Куда применить при вычислениях - неизвестно. Увеличим напряжение анодного источника до 165 вольт. Т.е., дадим приращение ∆Ua=165-150=15 В Под воздействием приращения напряжения произойдёт приращение анодного тока с 5 до 7 мА: ∆Ia=7-5=2 мА Если возьмём отношение этих относительных приращений, получим величину: Ri=∆Ua/∆Ia =15/2= 7,5 В/мА. Поскольку В/мА=кОм, то есть Ri выражается в единицах сопротивления, то величине Ri дали название ВНУТРЕННЕЕ СОПРОТИВЛЕНИЕ лампы. В нашем случае, для точки А: Ri=7,5 кОм. Таким образом имеем два похожих названия для разных величин, сильно отличающихся одна от другой как по сути, так и в числовом выражении. ЗЫ. Кстати, описанным под рисунком методом и измеряют Ri. ЗЫ.ЗЫ. Поскольку ВАХа лампы не является прямой линией, то, РАЗУМЕЕТСЯ, что в разных её точках величина Ri разная: чем ниже на линии ВАХ находится точка, тем Ri - выше. Всё семейство ВАХов для конкретной лампы состоит из математически неправильных кривых, и чем кривая ближе к оси Ia, тем меньше Ri в её точках при одном и том же значении Ia.

B_Aleko: Пермяк пишет: Попытаюсь ещё раз. Да, чисто теоретически, согласен. Можно ли где, найти первоначальный источник данного решения? И, по какому закону "сопротивление, проявлямое лампой в динамическом режиме", это движение рабочей точки по нагрузочной линии осуществляется? http://akadem-nauki.ru/dinamicheskij-rezhim-raboty-bpt-dinamicheskaya-xarakteristika/ только это такое же решение построенное по одним вычислениям, а сам закон?

Пермяк: Представим себе, что лампа полностью заперта. Ток через неё равен нулю. Тогда равен нулю и ток через Ra. Падения напряжения на Ra нет, и всё напряжение питания Ea приложено к аноду. Имеем точку с координатами (Ua=Ea; Ia=0), она лежит на оси напряжений. Теперь представим себе, что лампа полностью открыта. Через неё идёт ток теоретически максимальный, ограниченный только сопротивлением Ra: Ia.мах=Еа/Ra. Имеем вторую точку, с координатами (Ua=0; Ia=Ia.maх). Соединяем полученные две точки прямой линией (нагрузка активная), получаем траекторию перемещения рабочей точки. Таким образом, раб. точка перемещается по линейному ЗАКОНУ... э-э ... в данной системе координат

B_Aleko: "Теперь представим себе, что лампа полностью открыта....", по теоретическому вычислению, да. но практически этого никогда нет. линия Ra должна иметь совокупность и пропорцию Амплитуда-время. вот на графике видно, что красная линия нагрузки это совокупность и пропорция Амплитуда-время. А применение величины Ri по отрезку А-С, вносит искажение по времени.

Пермяк: B_Aleko пишет: Это справедливо при нулевом потенциале сетки и катода. И в этом случае, Ri = (Ua=Ea)/(Ia=0) =290k. Если сразу задать смещение, то этих величин не будет Ну что Вы такое пишете! Мы строим нагр. прямую для переменного тока, а катод соединён с землёй накоротко по перем. току через конденсатор Ск. Падающее на Rк ПОСТОЯННОЕ напряжение в построении не участвует, просто надо после всех вычислений добавить к Еа падающее на Rк постоянное напряжение.

B_Aleko: только переменного тока не будет без постоянных составляющих, которые и задают режим переменному току. вот потому и интересуюсь, откуда взялось вычислении величины Ri?

Пермяк: B_Aleko пишет: олько переменного тока не будет без постоянных составляющих, которые и задают режим переменному току. Не надо писАть банальных лозунгов, на моём последнем графике отражены ВСЕ постоянные составляющие. Только Вы их не видите. И что показывает Ваш последний график? Что такого нового и неожиданного? Просьба: читайте и обдумывайте свои посты перед их отправкой!

B_Aleko: скажет кто, где взять источник: откуда взялось вычислении величины Ri?

Ученик: Вся тема - троллинг.

B_Aleko: Ученик пишет: Вся тема - троллинг и где вы видели провокацию с моей стороны? если что то новое исследуете и изучаете, разве вы не рассматриваете возможные аспекты решений? или как все, под копирку. может и в жизни так же, что господин дал, то и правильно? сколько всего нового кругом. разве не пытливые люди это придумали? так бы и сидели в каменном веке. не нравится - не читайте.

Ученик: B_Aleko пишет: и где вы видели провокацию с моей стороны? Троллинг - более широкое определение, чем примитивная провокация... если что-то новое исследуете и изучаете Это "новое" (для Вас, возможно) давно исследовано и изучено, надо лишь учебники почитать.разве не пытливые люди это придумали Пытливые люди интересуются, спрашивают, читают литературу, вникают в ответы... а не выдумывают свои определения, ещё и упорствуют в них. Чего стоит очередное открытие в электротехнике и математике: B_Aleko пишет: Это справедливо при нулевом потенциале сетки и катода. И в этом случае, Ri = (Ua=Ea)/(Ia=0) =290k.Оказывается, 290В можно разделить на "0"мА, и при этом получить 290 килоом! Во как! Это и есть обещанная в стартпосте "масса интересных вычислений по этой теме и другим"? По Вашей писанине может быть два вывода: или Вы неспособны понять самых простых вещей, или Вы - тролль.

ALSS: B_Aleko пишет: А применение величины Ri по отрезку А-С, вносит искажение по времени. О каких "искажениях по времени" может идти речь в безинерционной системе?! Только не упоминайте нанопикосекунды (нано - слово затасканное и подорванное Сколковым). Я перестал под влиянием Пермяка писать о своем отношении к "выводам" топикстартера, но Ученик абсолютно прав. Вместо того, чтобы сделать за это время несколько макетов усилителей, послушать музыку и определиться в своих предпочтениях... впрочем, когда-то было сказано, что русский человек может сделать все. Если не будет задумываться об устройстве мироздания.

Пермяк: Ученик пишет: Оказывается 290В можно разделить на "0"мА, и при этом получить 290 килоом! Во как!Ну, обмишурился товарищ, с "новаторами" это частенько случается, а Вы - сразу за руку его хватать... :) B_Aleko пишет: скажет кто, где взять источник: откуда взялось вычислении величины Ri?А.М. Бонч-Бруевич. Применение электронных ламп в экспериментальной физике. М.,1956., стр. 68-71. Здесь попроще: Р.Лэнди и др. Справочник радиоинженера, 1961, стр. 39.

B_Aleko: ALSS, Вы думаете, я просто занят одними вычислениями? ошибаетесь. жаль только то, что у меня нет осциллографа и спектроанализатора, или спектролаба. по звучанию отличия ощутимы. при сборке "стандартным методом, наугад (Ra от величины 1...10Ri)" - ну звучит, ну относительно громко. нет обоснованности значения (Ra от величины 1...10Ri). отсюда и нет прямой зависимости от нагрузки.

B_Aleko: Пермяк пишет: А.М. Бонч-Бруевич. Применение электронных ламп в экспериментальной физике. М.,1956., стр. 68-71. а я и не знал, что у меня есть способность к математическим вычислениям. Давайте так, слегка подведу к истине. (мю), величина не постоянная, как утверждено - это статический, то есть максимальный параметр лампы при Ia=const., это сказано: Рейх. Теория и применения электронных приборов. стр. 81.; его значение Uвх=м. то есть (дельта), не "d", (дельта)=мю. Чем больше входной сигнал по амплитуде, тем больше мю, но ограничен он только (дельта)Ug. стр. 83. Далее, в книге: А.М. Бонч-Бруевич. Применение электронных ламп в экспериментальной физике. М.,1956., стр. 70., обратите внимание на формулу 1.87, из нее вытекают следующие 1.88 и 1.89. Обратите внимание на значение Uc=const. Вот, как сказал Иисус: "Имеющий глаза, да не видит...", или по нашему: "Смотрим в книгу, видим фигу". Эта величина "Uc=const" работает только в самих ее пределах!, она устанавливает правила для значений Ua в соотношении с Ia, не БОЛЕЕ! Кто ввел измерение значения Ri, в теорию Бонч-Бруевича???

Пермяк: B_Aleko пишет: (мю), величина не постоянная При первом же взгляде на семейство ВАХ любой лампы любому должно быть ясно, что все три параметра - S, мю, и Ri - величины НЕ постоянные. Но для этого, как минимум, надо знать, как они определяются. Вам очень долго, на 9 страницах, пытались объяснить, что такое эти "дельты", но Вы никак не хотели это понимать. Есть ТЕОРИЯ, и есть практические МЕТОДЫ РАСЧЁТА, основанные на теории. Методы расчёта во многих случаях бывают сознательно упрощены для уменьшения трудоёмкости расчётных работ. Погрешность в этих случаях (с учётом разброса параметров ламп) как правило, бывает вполне достаточной для производства аппаратуры, почему и применяются инженерами, и не только ими. ЗЫ. Это хорошо, что Вы нашли книгу Г.Дж.Рейха, вот посмотрите в ней, как надо рисовать вид сигнала во времени на графике: http://shot.qip.ru/00Qqtx-6gbw3dFgI/ Только рисунок этот - единственно для пояснения. Потому что, глядя на нагрузочную прямую триода, и без того видно, что правая полуволна тока всегда короче левой.

B_Aleko: а что же мне здесь все доказывали, что "мю" - постоянна и не меняется? да же график есть: Вот если бы потом дописали: при выбранных значениях режима лампы, тогда бы все стало на места.

Пермяк: B_Aleko пишет: а что же мне здесь все доказывали, что "мю" - постоянна и не меняется? Никто Вам такого не писАл. Я, например, постоянно в постах писал: для рабочей точки, для выбранной точки, для рассматриваемой точки. А на стр. 9, пост 4068, выложил даже график из ж. РАДИО для 6н3п с изменениями параметров лампы. Только Вы не захотели его осознать. http://f5.s.qip.ru/q7I7mMvP.png

B_Aleko: Да, видел. но все таки утверждения что она постоянна - были.

Пермяк: B_Aleko пишет: но все таки утверждения что она постоянна - были. Покажите, где. .

B_Aleko: Почему в радиотрактах приемников, при нагрузке только на индуктивность с малым сопротивлением, применяются пентоды? ведь там дело в этой же величине Ri. ,...а если триод, то там добавочное сопротивление?

Пермяк: Индуктивное сопротивление катушки на радиочастотах достаточно велико. На добавочном активном сопротивлении (резисторе) потеряем часть напряжения, развиваемое лампой.

B_Aleko: Пермяк пишет: Индуктивное сопротивление катушки на радиочастотах достаточно велико. а как же режим по постоянному току? катушка имеет очень низкое сопротивление и от этого мал ток. но что бы этот ток был, нужно приложить достаточно низкое входное напряжение. иначе лампа "умрет" при большем напряжении раскачки. верно? может не так разъясняюсь?

Пермяк: B_Aleko пишет: а как же режим по постоянному току? Так же, как и в трансформаторном каскаде-драйвере. Есть и каскады ЗЧ с дросселем в аноде.

B_Aleko: Пермяк, у вас был пост №4110, там график с линиями А и С. можете сказать величины Ri, dIa, dUa? кое что проясняется по книге Дж. Рейха. заранее благодарен. Так все же, какой правильный расчет Ri на графике?

Пермяк: Величины dUa и dIa - это дифференциальные (бесконечно малые) приращения напряжения и тока анода в точке А. Поскольку бесконечно малые отрезки на графике отразить невозможно, пользуются отрезками реальной длины, более-менее удобной для построения. Таков, например, отрезок АС на ВАХе, и отрезки его проекций на оси координат, которые в отличие от дифферециальных величин, обозначают греческой буквой "дельта": ∆Ua и ∆Ia. На графике пост 4110 http://hiend.borda.ru/?1-13-0-00000496-000-180-0#216 выполнено одно из возможных построений для определения Ri в конкретно выбранной точке А. Под графиком - в тексте - расчёт. Прочтите внимательно! Повторюсь: это не теоретические выкладки, а практический расчёт.

B_Aleko: Начну с "хвоста", так как рыбу чистят именно с хвоста. а где начало, "голова", история об этом умалчивает. Только прошу, без "разных" высказываний. Как смогу, буду объяснять, так как не владею математическим языком. Книга немецкого инженера, проживавшего в америке: Werner W. Diefenbach, 1954г. А вот график: , при выборе параметра Ri, исходят из точек А и С, но так как по формуле Ri=∆Ua/∆Ia, получается, что это точки ВС. И вычисляется этим образом по книге R(штрих), которое равно анодному сопротивлению. а Ri, не что иное, как Ri=Ua/Io (по книге). Исходя из данных, могу предположить, что Ri, не что иное как: изменение участка цепи внутреннего сопротивления лампы, при изменении действующих приращений переменных составляющих тока и напряжения. И все дело в том, что книг-оригиналов Бонч-Бруевича нет. Только начиная с 1960гг. и то, если присмотреться и прочитать между строк, становится очевидным, что кто то хорошо отредактировал данные его труды. А что касается трудов: Рейх. Теория и применения электронных приборов. 1948-1949гг. стр. 84. дифференциальное уравнение., то вполне может быть закономерностью с учетом времени, после выступления Черчиля на конгрессе.

B_Aleko: Далее. Дело в том, что немцы всегда отличались своей скурпулезностью и педантизмом. Принимаем во внимание тот факт, что значение Ri будет равно отрезку линии сопротивления Ra, смотрим второй график,где видно, что дельты Ri равны Ra в точках DE. :

B_Aleko: Ближе к сути: Добавим к графику еще линии смещения и дополнительные точки статических параметров: Становится почти очевидным, что статические параметры, точки G,J,K,L., для расчетов нам не нужны. Так как если брать их во внимание, то значения Ri, Ra будут значительно отличаться от расчетных. далее...

B_Aleko: книга Горшков, Справочник Радиолюбителя, 1937г. отрывки из книги: где сказано, что величина Ri - внутреннее уравнение лампы, но никак не внутреннее сопротивление по переменному току. И такое определение, что это "внутреннее сопротивление по переменному току" ни где не встречается в технической литературе до 1937 года.

Stan Marsh: B_Aleko пишет: где сказано, что величина Ri - внутреннее уравнение лампы Там это не сказано, читайте внимательно. Внутреннее уравнение лампы: μ = S*Ri

B_Aleko: Отсюда: Ri, вычисляется из отрезка АС или АВ (dU/dI), или полное приращение из отрезка ВС (∆Ua/∆Ia). последнее и является Ra. Stan Marsh пишет: Там это не сказано, читайте внимательно. Внутреннее уравнение лампы:Там не сказано, что оно является внутренним сопротивлению лампы по переменному току. И ещё вопрос. Из курса геометрии: Отрезок — это часть прямой линии, которая ограничена двумя точками (концами отрезка). У отрезка есть и начало, и конец. Длина отрезка — это расстояние между его концами. Как может половина отрезка являться полным сопротивлением лампы? Существует такая величина как Ro - величина внутреннего сопротивления постоянной составляющей режима покоя при заданных величин смещения. но куда ее применить? никто не знает. Этому значению есть прямое применение в решении параметров ламп. но об этом уже завтра.

Пермяк: B_Aleko пишет: Существует такая величина как Ro - величина внутреннего сопротивления постоянной составляющей режима покоя при заданных величин смещения. но куда ее применить? никто не знает. Так уж и никто? Более правильный перевод на русский технический язык: Rо - сопротивление лампы постоянному току в точке покоя, выбранной при заданных Ucм и Ua. И Вам уже неоднократно писали, что эта физ. величина на практике не нужна (мой пост 4110). Там не сказано, что оно является внутренним сопротивлению лампы по переменному току. Термины "по постоянному току" и "по переменному току" - не точные, при рассмотрении работы лампы они примененяются для упрощения. Лично Вас они просто сбивают с толку. Более точно надо говорить: "в статическом режиме", "в динамическом режиме". Т. е. при отсутствии усиливаемого сигнала, и при его наличии.Как может половина отрезка являться полным сопротивлением лампы?Ну, во-первых, Эта ПОЛОВИНА отрезка не ЯВЛЯЕТСЯ, а всего лишь отображает сопротивление. А во-вторых, и я Вам писал (пост 4110), да и на страничке в Вашем посте 215 сказано, что Ri - это отношение ИЗМЕНЕНИЯ анодного напряжения к ИЗМЕНЕНИЮ анодного тока. Тут, друг мой, надо смотреть с точки зрения математики. В системе координат, где гориз. ось - напряжение Uа, а вертикальная ось - ток Iа, сопротивление R выражается не ДЛИНОЙ отрезка, а отношением проекций отрезка на оси координат. Поэтому, если это отрезок ПРЯМОЙ линии, то - независимо от его длины - отношение проекций всегда одинаковое: R=ΔUa/ΔIa Очень жаль, что Вы этот важнейший момент до сих пор не поняли. Трудно, конечно, "не для средних умов..." (с) Попробую показать на рисунке. Пусть имеем некую прямую в указанной системе координат. Выберем отрезок АВ, и определим сопротивление, которое отображает этот отрезок, а заодно - и вся прямая: И гораздо всё будет интереснее, если там будет не прямая, а кривая, да ещё математически неправильная, каковыми являются ВАХи ламп.

B_Aleko: Пермяк пишет: И гораздо всё будет интереснее, если там будет не прямая, а кривая, да ещё математически неправильная, каковыми являются ВАХи ламп. Да, все гораздо будет интереснее дальше. И как вы верно заметили про значение R=ΔUa/ΔIa (не Ri) на графике. Точка В - это рабочая точка. Что это значение R=ΔUa/ΔIa есть на самом деле? И Вам уже неоднократно писали, что эта физ. величина на практике не нужна (мой пост 4110). не спешите с выводами. все по порядку.

B_Aleko: Продолжаю. Есть одно, большое замечание, по тому, как строят графики и указывают на них входные и выходные величины. к чему это? Все абсолютно уверены, что лампа делает фазовый сдвиг входного сигнала. Я думаю, что этого не происходит, судя по данным ниже графиков. Может кто поможет потом точно разобраться. Вот здесь можете забросать меня камнями, удалить все мои сообщения и прочее............................. Конечно можете, но наберитесь терпения. Так это тоже не мной придумано. Это скан из книги: Как могу, объясню. Дело в том, по представлениям что рисуем, входной сигнал - мы размещаем ниже оси абсцисс, ток - справа оси ординат, а выходной сигнал - на "биссектрисе" линий смещения. Может кто то объяснит, как возможно в таком случае, что линия сеточного смещения является "зеркалом" для вектора амплитуды. И вдобавок увеличивать значение вектора напряжения выше точки Ua.max? А где же анодная нагрузочная линия? Вот она то и выполняет распределение (стекания, не отражения) тока-напряжения на своем отрезке, обусловленном точками А и В. (ну не математик я. как могу...) график 4: если я окажусь не прав о фазовом сдвиге, заранее простите. Но в том, что на этом графике точно указаны векторы направления сигналов - факт. Обратите внимание на значение нагрузочной линии: RL=25000 Ом.

B_Aleko: смотрим на американский график. его значения: Ua.min = 92.5 Ua.max = 217.5 Ia.max = 0.0103 Ia.min = 0.0053 ΔUa = Ua.max - Ua.min = 217.5 - 92.5 = 125 ΔIa = Ia.max - Ia.min = 0.0103 - 0.0053 = 0.005 dUa = ΔUa / 2 = 62.5 dIa = ΔIa / 2 = 0.0025 Вычисляем анодное нагрузочное: Ra = ΔUa / ΔIa = 125 / 0.005 = 25000 Найдем рабочую точку, так как на графике ее точных значений нет. Uo = Ua.max - dUa = 217.5 - 62.5 = 155 = Uo = Ua.min + dUa = 93.5 + 62.5 = 155 Io = Ia.max - dIa = 0.103 - 0.0025 = 0.0078 = Io = Ia.min + dIa = 0.0053 + 0.0025 = 0.0078 И вот последнее недостающее звено: Ro = Uo / Io = 155 / 0.0078 = 19872 Определяем напряжение питания: Ea = (Ro + Ra) x Io = (19872 + 25000) x 0.0078 = 350.0016 Все величины в формулах - по постоянному току! Смотрим на график..... Только одно но. Эти значения абсолютно справедливы для паспортных (заводских) ВАХ. Проверено не только на 6Н3П , но и на лапах 6Н2П, 6Н8С, 6Н9С, 6Н23П, 12ax7 На графиках снятых "вручную", достоверность может оказаться плохой. Как то так, СПАСИБО.

B_Aleko: И последнее дополнение: Если я хочу усиление =20, выбираю 6Н23П РТ со значениями: Uo = 70 Io = 0.0095 Imax = 0.014 Imin = 0.0065 Umax = 90 Umin = 50 Вычисляю: Ro = 7368 Ra = 5333 Ea = 120 график для сравнения

Stan Marsh: B_Aleko пишет: Если я хочу усиление =40, выбираю 6Н23П Не получится. μ не позволит.

ALSS: B_Aleko пишет: Все абсолютно уверены, что лампа делает фазовый сдвиг входного сигнала. Чиво?! В интересующей нас области звуковых частот лампа абсолютно безинерционный прибор. Чего не скажешь о каскаде, в котором она используется. Не обобщайте свое незнание на весь мир и тем более не выдавайте свое "прозрение" за мировое откровение.

B_Aleko: Уважаемые форумчане! Это НЕ мои собственные мысли! Это было придумано почти сотню лет назад. Если вы считаете все описанное "бредом, ахинеей, вымыслом", прошу, без оскорбления. Попробуйте сами разобраться в имеющемся и решить для себя, надо это вам или нет! Мне НАДО! потому, что это правильно. СПАСИБО.

Ученик: B_Aleko пишет: Это НЕ мои собственные мысли! Это было придумано почти сотню лет назад. Учебники, которые Вы предъявляете - да. Но выводы, которые Вас "озаряют", неверные. И самое плохое - Ваше нежелание прислушаться. В конце-концов, если Вы так уверены в своей правоте, получается, что оппонирующие Вам ничего не понимают? Все разом? Обычно бывает по-другому: если у кого-то "все вокруг дураки", это значит, что дурак всего один.

B_Aleko: Ученик пишет: Все разом? Вы за всех не говорите. P.S. Пусть оценят, просчитают. А там примут решение. до вечера, с детьми еду отдыхать!

Ученик: B_Aleko пишет: Вы за всех не говорите Сторонников Ваших выкладок не видать.Пусть оценят, просчитают. Так Вы уже всё "просчитали", и даже сделали вывод:... это правильно. Зачем Вам другие мнения?

Jaster: Почитал тему, вроде многое понятно, но все же есть вопросы В частности, допустим я выбрал рабочую точку для лампы в трансформаторном каскаде. А что в таком случае будет при изменении анодной нагрузки? Сама точка же не меняется, если изменилась только нагрузка?

Бокарёв Александр: подрежет вашу точку с одной стороны.Точку мы выстраиваем на конкретной нагрузке по симметричному ограничению и далее по наичистому спектру.

Jaster: Сорри, я имел в виду увеличение нагрузки относительно оптимальной.

Бокарёв Александр: Ограничение сдвинется в другую сторону

Jaster: Бокарёв Александр Странно, почему тогда расчет по ВАХ показывает минимальные искажения, просто мощность меньше?

U.L.F.: Jaster пишет: Странно, почему тогда расчет по ВАХ показывает минимальные искажения, просто мощность меньше? Почему же минимальные? Он показывает в основном искажения по второй гармонике и не факт, что они минимальные в выбранной Вами точке. Вам к истокам нужно.Такое впечатление, что Вы немного не понимаете как это всё работает. Сюда: http://ulfdiysound.ucoz.ru/load/nachinajushhemu_audio_samodelshhiku/3

Buran81@inbox.ru: Jaster пишет: А что в таком случае будет при изменении анодной нагрузки? Сама точка же не меняется, если изменилась только нагрузка? В трансформаторном каскаде, конечно, от смены нагрузки на вторичке положение самой точки покоя на ВАХ не меняется. Ток и напряжение покоя на аноде, как были, так и останутся. В резистивном каскаде смена анодного резистора меняет и положение раб. точки.

Jaster: Вот. теперь сомнения рассеялись. :) Спасибо всем.



полная версия страницы